[ /tv/ /rf/ /vg/ /a/ /b/ /u/ /bo/ /fur/ /to/ /dt/ /cp/ /oe/ /bg/ /ve/ /r/ /mad/ /d/ /mu/ /cr/ /di/ /sw/ /hr/ /wh/ /lor/ /s/ /hau/ /slow/ /gf/ /vn/ /w/ /ma/ /azu/ /wn/ ] [ Main | Settings | Bookmarks | Music Player ]

No.144684 Reply
File: __chibi_mathematicians___by_eulerstalker-d5iz9c6.png
Png, 705.87 KB, 2079×1300 - Click the image to expand
edit Find source with google Find source with iqdb
__chibi_mathematicians___by_eulerstalker-d5iz9c6.png
Наша кафедра продолжает свою работу. Здесь мы обсуждаем математику, а также иногда отвечаем на простые вопросы доброанонов.
Тред обучения математике: >>124265

Библиотека: http://gen.lib.rus.ec

Калькуляторы:
http://online-integral.ru/
http://www.wolframalpha.com/

Предыдущий: >>141253
>> No.144734 Reply
File: Screenshot_2015-0...
Png, 8.63 KB, 538×230
edit Find source with google Find source with iqdb
Screenshot_2015-09-03_22-05-29.png
File: Screenshot_2015-0...
Png, 17.79 KB, 242×249
edit Find source with google Find source with iqdb
Screenshot_2015-09-03_22-05-38.png

Поясните первокурснику-гуманитарию геометрию Лобачевского: почему на рисунке 1 гиперболические линии называют прямыми? Почему если на рисунке 2 прямые пересекаются, то постулируется, что они параллельны?
>> No.144737 Reply
>>144734
Первый вопрос: они называются прямыми потому, что мы можем их так назвать. Геометрия Лобачевского есть некоторый proof of concept: доказывается, что мы можем построить некую "геометрию", в которой будут существовать объекты, которые мы будем условно называть "точками" и "прямыми". Для этих объектов будут заданы некие отношение("параллельность", "лежать на", "пересекаться"), таким образом, что для этих объектов и отношений будут выполнены все аксиомы евклидовой геометрии, кроме постулата о параллельных. Этим доказывается то, что без постулата о параллельных можно существовать, просто получиться другая геометрия. При этом объекты, которые мы называем "прямыми" или "точками" не обязаны таковыми являться - "прямая" вполне может быть и окружностью(как в сферической геометрии) или, опять же, куском гиперболы. Собственно, с тем же успехом мы могли бы говорить о "столах" и "стульях".
Второй вопрос: они пересекаются, но все они параллельны линиям, отмеченным на рисунке жирным. Это показывает невыполнение постулата о параллельных: через одну точку(пересечение двух жирных линий) проходит множество "прямых"(хорд окружности в данной модели), параллельных(в данной модели - не пересекающихся внутри круга) с данной жирной, что невозможно в обычной модели евклидовой геометрии.
>> No.144774 Reply
File: test.pdf
Pdf, 0.04 KB, 595×842 - Click the image to get file
test.pdf
>>144684
Наконец-то решил.
>> No.144778 Reply
>>144774
Решил ты неправильно, что легко можно понять, подставив вместо p какое-нибудь число. Например, при р=1 финальное выражение существенно больше начального. Половина преобразований в твоём решении вообще не имеет смысла. С чего ты, например, взял, что можно просто взять и возвести числитель дроби в квадрат(первый переход)? А главное, тебе же были даны указания о том, что делать с этим упражнением. Почему ты им не следуешь, а вместо этого занимаешься хуйнёй? Если бы ты последовал данному тебе совету и тщательно разобрал формулы сокращённого умножения, то проблем у тебя этот номер бы не вызвал.
>> No.144794 Reply
>>144778
Посмотрел что напечатал и пиздую записываться в списки больных дисграфией.
На бумажке решено правильно. Видимо надо просто не выебываться и писать от руки и фоткать.

В знаменателе работает формула квадрата суммы.
В числителе - формула суммы кубов.

Спасибо анон за долготерпение.
И прости идиота.
>> No.144795 Reply
File: IMG_0041.jpg
Jpg, 919.81 KB, 2592×1944 - Click the image to expand
edit Find source with google Find source with iqdb
IMG_0041.jpg
>>144794
Всю жизнь забивал на математику, раскочегариваться непросто. Если совсем-совсем достал, могу не забивать борду своим спамом же...
>> No.144798 Reply
>>144795
А вот тут правильно и осмысленно.
> Видимо надо просто не выебываться и писать от руки и фоткать.
Вот-вот.
> Если совсем-совсем достал, могу не забивать борду своим спамом же...
Не думаю, что кто-то будет против, тут и в наиболее оживлённое время полторы калеки.
>> No.144802 Reply
>>144613
Бамп.
>> No.144812 Reply
>>144802
Может, здесь спросишь что-нибудь?
>> No.144831 Reply
File: 1320986356008.jpg
Jpg, 197.02 KB, 849×1181 - Click the image to expand
edit Find source with google Find source with iqdb
1320986356008.jpg
>>144812
Да. Где используется алгеом и почему вербитобляди так надрачивают на все эти плюшки?
>> No.144832 Reply
>>144831
Алгеом - абстрактная дисциплина. Приведи четыре примера абстрактных дисциплин, которые где-нибудь используются.
>> No.144833 Reply
>>144831
> вербитобляди так надрачивают на все эти плюшки
Они же вроде надрачивают на алгебраическую топологию, а не на алгебраическую геометрию.
>> No.144834 Reply
>>144832
> Приведи четыре примера абстрактных дисциплин, которые где-нибудь используются.
Теоркат, алгебра, топология и теория типов (хотя последнее ближе к информатике).
>> No.144835 Reply
>>144832
Хм. Диффгеометрия, теоркат, теория чисел, банальный линал?
>> No.144836 Reply
>>144834
Теоркат не используется нигде. Алгебра, за исключением линейных уравнений, не используется нигде. Топология не имеет приложений. Теория типов - мёртвая вещь.
>> No.144837 Reply
File: 135334616456.jpg
Jpg, 555.96 KB, 1600×1200 - Click the image to expand
edit Find source with google Find source with iqdb
135334616456.jpg
>>144833
Мне казалось, сейчас самая модная отрасль - алгеом. Возможно я слегка ошибся.
>> No.144839 Reply
File: IMG_0043.jpg
Jpg, 801.11 KB, 1801×2121 - Click the image to expand
edit Find source with google Find source with iqdb
IMG_0043.jpg
>> No.144841 Reply
>>144831
Теория кодирования, например. Ни одна крупная банковская операция не проходит без использлвания шифров, построенных на фактах алгебраической геометрии.
>> No.144843 Reply
>>144836
> Теоркат не используется нигде.
Информатика.
> Алгебра, за исключением линейных уравнений, не используется нигде.
ruwiki://Код_Рида_—_Соломона
> Топология не имеет приложений.
Удивительные истории.
> Теория типов - мёртвая вещь.
http://homotopytypetheory.org/
>> No.144844 Reply
>>144836
> Топология не имеет приложений.
Ололо. Квантовая механика(активнейше использующая топологические векторные пространства) и теория относительности(гладкие многообразия) - это не приложения?
>> No.144845 Reply
File: 13181068053577.jpg
Jpg, 271.88 KB, 800×650 - Click the image to expand
edit Find source with google Find source with iqdb
13181068053577.jpg
>>144841
То есть, алгеом имеет касательство к криптографии?
>> No.144847 Reply
>> No.144848 Reply
>>144839
Не вижу ошибок.
>> No.144849 Reply
>>144843
Теоркат не используется в информатике никак. Даже в функциональном программировании никто не знает о лемме о змее.
> ruwiki://Код_Рида_—_Соломона
Обычные линейные уравнения.
> homotopytypetheory
Эти ребята до сих пор пытаются доказать свою полезность хоть кому-то. Хотя пропиарились.

>>144844
> топологические векторные пространства
Топология нужна там, только чтобы производную определить. Аксиомы отделимости, вес топологического пространства, линделёфовы пространства, паракомпакты, критерий Бинга - о таком никто в квантовой механике и не слышал. То есть общая топология никому не нужна.
> гладкие многообразия
Когомологии Чеха? Когомологии Дольбо? Гомологии Бореля-Мура? Ха.
>> No.144850 Reply
File: 13231122090631.png
Png, 453.30 KB, 544×416 - Click the image to expand
edit Find source with google Find source with iqdb
13231122090631.png
>>144847
Спасибо, анон.

>>144849
Ну, будь я уверен, я бы вообще не задавал изначального вопроса. Так что может я и жестоко ошибаюсь. Однако...
Теоркат, кажется, примеряли на квантмех. http://arxiv.org/abs/0808.1032
Упомянутая диффгеометрия вовсю абузится в относительности.
Упомянутая теория чисел - опять в криптографии, вопреки чаяниям Харди о чистой и незамутнённой приложухой математике.
Упомянутый линал - дохрена где в физике.
>> No.144851 Reply
>>144850
> Теоркат, кажется, примеряли на квантмех
Всего лишь proof-of-concept, который авторы написали от безысходности.
> вовсю абузится в относительности
Мифы.
>> No.144852 Reply
>>144849
> Топология нужна там, только чтобы производную определить.
Ебанись просто. Секвенциальная компактность, теорема о замкнутом графике, продолжения по Фридрихсу и Фон Нейману неограниченных операторов - это все не топология?
> Когомологии Чеха?
Слабо относятся к гладким многообразиям, имеют больше примененря в абстрактной алгебраической геометрии - как теория когомологий для схем.
> Когомологии Дольбо?
Оператор Виттена и осцилляторное представление уравнений квантмеха.
> гомологии Бореля-Мура?
Я тоже знаю много страшных слов.
>> No.144853 Reply
>>144851
> Мифы
Т.е. диффгеометрия не используется в теории относительности, я правильно тебя понял?
>> No.144855 Reply
>>144852
> это все не топология?
Это детский лепет какой-то, а не топология. Простейшие следствия самых первых определений, маленькая льдинка в бескрайнем океане.
> много страшных слов.
И даже эти простые вещи ты называешь "страшными" - то есть подсознательно ты прекрасно понимаешь, что они не имеют приложений.
>> No.144856 Reply
>>144853
Совершенно правильно. Дифгеометрия не используется нигде, за исключением, возможно, до смешного крошечных её кусочков.
>> No.144858 Reply
>>144856
А в чем твой месседж-то состоит? Что значительная часть математики не применяется в других науках и используется только в самой математике?
Так я это знаю, например. И меня это не смущает, так как математика иначе не могла бы развиваться.
> И даже эти простые вещи ты называешь "страшными" - то есть подсознательно ты прекрасно понимаешь, что они не имеют приложений.
Это уже демагогия. На одну из теорий когомологий я тебе назвал вполне конкретные приложения, две оставшиеся практически не применяются для гладких многообразий - Чех для абстрактных пространство с пучками, Борель-Мур - в теории пересечений.
>> No.144859 Reply
>>144858
> А в чем твой месседж-то состоит? Что значительная часть математики не применяется в других науках и используется только в самой математике?
Представим, что есть сорок книг по шестьсот-семьсот страниц, принадлежащих одной науке. Есть также одна тонкая книжка в двести страниц и пять журнальных статей, принадлежащих другой науке. Во втором наборе литературы используется примерно пятнадцать абзацев из первого набора литературы (и для понимания этих абзацев не обязательно читать все входящие в него сорок книг).

Можно ли охарактеризовать эту ситуацию: первая наука используется во второй науке? Я думаю, что нельзя.
>> No.144860 Reply
>>144859
> пятнадцать абзацев
Только их далеко не пятнадцать.
>> No.144861 Reply
>>144860
Пятнадцать. От силы двадцать.
>> No.144862 Reply
>>144861
Вся http://imperium.lenin.ru/~verbit/MATH/programma.html нацелена лишь на подготовку специалистов для одной известной физической теории.
Это явно не пятнадцать абзацев, и не двадцать.
>> No.144863 Reply
>>144859
А ты не думай, у тебя это не очень-то удается. Если мерять науку на погонные метры("в физике используется только каждая десятая страница текста, написанного математиками" - это буквально твой аргумент), то доказать можно всё, что угодно. Химия не используется в фармацевтике, потому что химики пишут больше книг, чем нужно фармацевтам. Теория вероятностей не используется в статистике, потому что статистикам не нужны сто видов сходимости случайных величин. Биология не нужна в медицине, медики и так знают, как резать подопытных лягушек. Вообще никакая наука никогда не использовалась ни в какой другой. Доволен?
>> No.144864 Reply
>>144862
На самом деле это краткое изложение того, что Тифарет понимает под Core Mathematics.

>>144863
Именно в этом мой тезис, да. Ситуацию, когда в науке-1 используется крошечная часть науки-2, буквально капля в океане, правильнее называть не использованием, а другим термином, например "содержать следовые количества". Фармацевтика содержит следовые количества химии. Статистика содержит следовые количества науки о вероятностях. Математика содержит следовые количества логики. Физика содержит следовые количества математики. Химия содержит следовые количества физики. Биология содержит следовые количества химии. Медицина содержит следовые количества биологии.
>> No.144865 Reply
>>144864
Тогда о чем мы вообще говорим? Слово "использовать" беспокоит твой аутизм и ты хотел поспорить об этимологии? Не то, чтобы я был против, но в таком разрезе это разговор скорее социологический и философский, нежели математический. Ты мог бы подобрать для него более подходящий тред.
>> No.144866 Reply
Тем более, что без целого океана необходимых капель всё равно не получитью
>> No.144867 Reply
Скажите, могут ли существовать 2 зависимые случайные величины X, Y, такие, что их характеристические функции fx(t), fy(\tau), f{x, y}(t, \tau) (характеристические функции X, Y и вектора (X, Y) соответственно) удовлетворяют условию: f{x, y}(t,t) = fx(t)fy(t).
Сначала пытался действовать как в переформулировке независимости в терминах характеристических функций. Не получилось. Точнее, получилось лишь сказать, что если функции зависимы, то должен быть заряд на плоскости, зануляющий cos(t(x+y)) и sin(t(x+y)) для любого t.
Потом решил придумать пример. Вроде Y = -X, где X - случайная величина, почти наверняка равная единице, подойдет? А что-нибудь не дискретное может быть?
>> No.144868 Reply
>>144865
Мы обсуждаем это в правильном треде. Слова естественного языка, например русского, - это скрытые умозаключения. Я вполне обоснованно возражаю против идеи, что одна наука может использоваться в другой науке, и утверждаю, что науки могут лишь содержаться друг в друге в следовых количествах. Поэтому первый вопрос из поста >>144831 следует переформулировать так: "В каких науках в следовых количествах содержится алгебраическая геометрия?"
>> No.144869 Reply
>>144867
Простите, ноутбук сломался и набрать текст в пдф я не могу. Пытался использовать символы нижнего подчёркивания, как в техе, но их сожрала разметка.
как ужасно живется без комплуктера
>> No.144870 Reply
>>144869
Чтобы Ханабира не съедала символы подчёркивания и звёздочки, используй http://dobrochan.com/help/wakabamark же.
>> No.144871 Reply
>>144867
Могут существовать.
https://en.m.wikipedia.org/wiki/Subindependence - см. ссылки.
>> No.144872 Reply
>>144868
Мне совершенно не интересно спорить о терминологии. Можно говорить "гиперповерхность", можно "многообразие коразмерности один". Можно говорить "симметрическая группа", можно "группа перестановок". Хочешь говорить о следовых чем-то там вместо короткого и простого слова "использовать" - хер с тобой.
> Слова естественного языка, например русского, - это скрытые умозаключения.
Я не разделяю эту точку зрения.

Скажи, а ты не тот смешной чувак из /rf/, сетовавший на индикатриссу Дюпена в курсе дифференциальной геометрии? Слог похож.
>> No.144874 Reply
>>144872
> Я не разделяю эту точку зрения.
Почему? Она состоит в том, что слова не являются просто колебаниями воздуха. Они имеют некий неформальный смысл. Даже у синонимов смысловые оттенки различаются. Разве тебе это не очевидно?
> смешной чувак из /rf/
Я не считаю его смешным.
>> No.144877 Reply
>>144874
> Разве тебе это не очевидно?
Нет. В этом вопросе я следую Витгенштейну. Слово есть его употребление и больше ничего. Единственная причина, по которой ту же группу симметрий мы не называем группой биекций - историческая традиция. То же относиться и к слову "использовать" применительно к отношению между науками. Тем не менее, эта причина достаточно серьезна.
Если и считать, что за словами стоят чьи-то умозаключения, то это умозаключения не единичного человека, а растянутых во времени групп людей. Групповое мышление достаточно сильно отличается от мышления единичного человека, чтобы рассуждения вида "а давайте строить фразы, исходя из аристотелевой логики", не радотали.
> Я не считаю его смешным.
Это равносильно признанию. Может, перенесем этот разговор в твой уютный бложик, чтобы не заниматься оффтопом?
>> No.144878 Reply
>>144874
Что за чувак?
>> No.144879 Reply
>>144871
Спасибо, няша.
>> No.144887 Reply
File: 14383436688820.jpg
Jpg, 46.43 KB, 576×384 - Click the image to expand
edit Find source with google Find source with iqdb
14383436688820.jpg
Инженеру, Физику и Математику сказали зашкварить как можно больше народу на хате и при этом самому не зашквариться.
Запускают инженера. Он снимает штаны начинает кружиться на месте и ссать, стараясь задеть всех, ну его сразу сбивают с ног, пиздят и ссут на него всей хатой.
Затем заходит физик. Он начинает обьяснять, что все сделано из атомов, запах - частицы вещества и что таким образом все на хате имели контакт с мочой, петухами и вообще зашкварены ещё с рождения. Физика спрашивают, а зашкварен ли он? Тот отвечает положительно. Ну его пиздят и говорят что самозашквареные не могут людей шкварить.
Впускают математика. Тот сразу ныряет род шконку и орет. Инвернтирую пространство относительно шконоря! Вы все под шконкой!
>> No.144910 Reply
File: breakingnews.gif
Gif, 27.74 KB, 350×400 - Click the image to expand
edit Find source with google Find source with iqdb
breakingnews.gif
>>144849
Про лемму о змее не скажу, (она чем-то особенно интересна?)
но кое-что интересное можно найти даже в модном нынче Хаскелле, хотя тут я сам знаю не очень много.
https://hackage.haskell.org/package/category-extras
> Эти ребята до сих пор пытаются доказать свою полезность хоть кому-то. Хотя пропиарились.
Энивэй вопрос об основаниях математики никто не отменял, при чём тут полезность?
> То есть общая топология никому не нужна.
Как раз поэтому сейчас ей почти никто не занимается, а
основы общей топологии лежат в основаниях и дифференциальной геометрии, и алгебраической топологии.
Лет сто назад она позволила понять, какими бывают геометрические объекты, позволила выработать более современные подходы к геометрии.

>>144855
Что для тебя детский лепет? По-моему весьма логично, что передовые (условно) достижения науки трудно применять в другой науке, постольку поскольку человек применяющий, например, "математику А" к "физике Б" должен быть не только на переднем крае "физики Б", но и на переднем крае "математики А". Если ты посмотришь учебники пятидесятилетней давности по ОТО, то разумеется из геометрии ты не найдёшь там ничего сложнее связностей, тетрадного подхода Картана и прочего (что уже вылезает за рамки стандартного образования, например, на мехмате). Тем более в учебниках для начинающих, или для экспериментаторов, там от математики бегут как от огня. Но посмотри на не настолько известные современные иследования в области, например, струн и инстантонов, и ты увидишь вещи поинтересней (ещё в старых работах, 80-е, 90-е, если не изменяет память, мелькали и теорема Атьи-Зингера, и алгебраическая топология, и виртуальные расслоения, представления классических, аффинных алгебр, чего только не, не говоря уже о всей этой суете вокруг пространств модулей кривых). В конце концов почти всё, что есть в книжке современная геометрия так или иначе применялось ещё во время её написания в теорфизике, хотя, далеко не каждый теорфизик знает даже это. Ну и всё-таки немало филдсов за последнее время выдали за матфизические исследования.

Учитывая всё это про льдинку в океане звучит весьма поэтично, но, на мой взгляд, далеко от правды. Я бы сказал, что скорее как классика (в широком смысле слова) в литературе. Это некоторая база, вокруг которой обрастает наука. В этом ключе она движется. Когда нечто становится более понятным, и становится более ясным его смысл, оно зачастую применяется в других науках. Или теряется.
>> No.144924 Reply
File: face14352501612260.jpg
Jpg, 26.54 KB, 250×395 - Click the image to expand
edit Find source with google Find source with iqdb
face14352501612260.jpg
Матоны, а встречал ли кто-нибудь полное изложение задачи о классификации пар отображений где-нибудь, кроме как в Теории матриц Гантмахера?
>> No.144925 Reply
Анон, помоги с интегралом.
ʃ((x-sinx)/(1-cosx))dx
Если не трудно, напиши процесс решения, чтобы я потом смог сам такой решить.
>> No.144927 Reply
>>144925
Ссылка на калькуляторы в ОП-посте.
>> No.144933 Reply
>>144927
К сожалению не получилось решить интеграл (x-sin(x))/(1-cos(x))
>> No.144934 Reply
>> No.144958 Reply
Пусть внутри выпуклого четырехугольника ABCD существует такая точка М, что треугольники АМВ и СМD равнобедренные с углами 2pi/3 при общей вершине М. Докажите, что на плоскости найдется такая точка N, что треугольники BNC и DNA равносторонние.

Что-то я далеко не ушел в рассуждениях, да и мыльные они у меня, без обоснованные формальными выкладками. Ну и чтоб не просить рыбу каждый раз, поясните, из-за чего может быть так туго с геометрией? Просто опыта мало, дыры в знаниях? Со всеми остальными дисциплинами норм.
>> No.144962 Reply
>>144910
Я просто проходил мимо и захотел тебя поблагодарить за твой пост. Спасибо.
>> No.144967 Reply
>>144934
Запоздалое спасибо.
>> No.144973 Reply
Кто может помочь с переводом знаковых дробей в двоичный код как именно это делается в курсе, но чёт конечный результат хромает, а после этот код в обратный\дополнительный? Или хотя бы, для начала, подскажите как назад перевести дробь из двоичного кода в десятеричный.
>> No.144994 Reply
Очевидно, математика на доброчане не в почёте. Спасибо, я уже сам превосходно разобрался и могу кому-либо помочь, если будут вопросы. Обращайтесь.
>> No.144999 Reply
>>144994
Что делает тебя сделать такие выводы?
>> No.145024 Reply
Сап, анон помоги решить задачу:
Функцию g определено как: каждому натуральному числу поставлено в соответствие остаток деление числа на 4. И соответственно доказать что g(x) = g(x-4) для любого натурального числа.
>> No.145025 Reply
>>145024
там g(x+4)
>> No.145027 Reply
>>145024
Если x = 4a + b, то x + 4 = 4(a + 1) + b, и g(x) = g(x + 4) = b.
>> No.145080 Reply
Доброчан, нужна твоя помощь, после школьной математики у нас в институте пошли интегралы и дифференцирование, это то, что у нас идет по физике, а по математике идут всякие матрицы и интегралов там вообще не заметно. Так вот, есть ли краткая методичка, чтобы быстро разобраться с самыми основами? Мне бы пока на не очень сложном уровне, чтобы просто я смогу бы применять это все в физике, а то без этого я не могу решать задачи, а это влечет за собой долги и прочие нехорошие вещи.
Извиняюсь за неровный текст, очень сильно волнуюсь.
>> No.145081 Reply
>>145080
Методичка по чему? По физике, по интегралам или по матрицам?
Не волнуйся, в первый месяц первого курса рано думать о гипотетических будущих долгах.
>> No.145083 Reply
>>145081
Методичка по интегралам и дифференцированию, просто мы занимаемся уже такими вещами на физике, а я немного не въехал в эти интегралы, как и 90% моей группы и теперь не могу решать задачи по физике, но не могу их решать из-за того раздела математики, который я пока не понимаю. Хотелось бы на начальном уровне разобраться.
У меня просто от моего бессилия в физике руки опускаются, пока я думаю, что все упирается в математику, очень надеюсь, что это так, а как только начинаю думать, что может быть есть еще проблемы, то просто впадаю в депрессию.
>> No.145085 Reply
>>145080
Могу посоветовать Письменского Конспект лекций по высшей математике. Там очень просто расписывается основы, но сложные интегралы ты по ней не разгрызешь.
>> No.145086 Reply
>>145080
Могу посоветовать Письменского Конспект лекций по высшей математике. Там очень просто расписывается основы, но сложные интегралы ты по ней не разгрызешь.
>> No.145087 Reply
>>145085
Сложные пока не надо, пока хватит основ, спасибо большое! Сейчас попробую конспект этих лекций.
>> No.145097 Reply
File: ea3df4a4460eaff95cc1a32fb1acb78f.jpg
Jpg, 80.88 KB, 600×880 - Click the image to expand
edit Find source with google Find source with iqdb
ea3df4a4460eaff95cc1a32fb1acb78f.jpg
Суть такова. Есть три предмета: матан, дискретка, алгебра и аналитическая геометрия. И есть я — первокурсник. Не сказал бы, что совсем тупой, но некоторые вещи не понимаю в объяснении преподавателей, некоторые вещи они не говорят сами, случайно или намеренно.
Посоветуйте книг, чтобы не оставалось пробелов. Желательно не сильно сложным языком написанные, либо с введением, где поясняется как на человеческий перевести.
Рандомбака.
>> No.145098 Reply
>>145097
Каждый университет работает по своей собственной образовательной программе. Каждый преподаватель университета имеет свои собственные представления о предмете. Предметы с одним и тем же названием в разных вузах сильно отличаются друг от друга. "Дискретная математика" в МГУ, "Дискретная математика" в СПбГУ и "Дискретная математика" в книге Андерсона - три мало похожих друг на друга дискретных математики. Поэтому нет книги, в которой бы было написано в точности то, что вещает тебе твой лектор. Учитывай это.

Стандартный набор книг для первого курса:
1. "Краткий курс высшей математики", Демидович.
2. "Конспект лекций по высшей математике", Письменный.
3. "Математический анализ", Зорич.
4. "Дискретная математика и комбинаторика", Андерсон.
5. "Курс алгебры", Винберг.

По аналитической геометрии добавлю, вдовесок к Демидовичу и Письменному, две книжки с одинаковым названием "Аналитическая геометрия", одну из которых написал Погорелов, другую - Постников. Написаны плохим языком, их сложно читать, но содержание более-менее покрывает то, что подразумевается под аналитической геометрией в большинстве университетов.
>> No.145100 Reply
>>145098
Добавлю, однако, что книга Андерсона среди многих не очень ценится из-за большого количества ошибок, к которым добавляются ещё и возможные недопонимания из-за корявого перевода.
>> No.145101 Reply
>>145083
http://mathprofi.ru/integraly_primery_reshenij.html и несколько следующих уроков. Минимум теории, максимум практики, за пару вечеров 100% освоишься.
>> No.145102 Reply
File: Gelbooru-1442510-1girl-armpits-arms_up-bare_legs-b.jpg
Jpg, 234.08 KB, 1280×906 - Click the image to expand
edit Find source with google Find source with iqdb
Gelbooru-1442510-1girl-armpits-arms_up-bare_legs-b.jpg
>>145098
> Предметы с одним и тем же названием в разных вузах сильно отличаются друг от друга
Жаль, я думал хоть математику могли бы нормально стандартизировать. А так у меня подозрения, что нам её дают крайне слабо.
За книги спасибо, посмотрю, сравню с тем что изучаем.

>>145101
Спасибо.
>> No.145103 Reply
File: IMG_1419.jpg
Jpg, 187.31 KB, 814×772 - Click the image to expand
edit Find source with google Find source with iqdb
IMG_1419.jpg
>>145101
Спасибо большое, правда сегодня я решаю огромное кол-во матриц и матричных уравнений, но что поделать, надо решить, чтобы понять как я усвоил материал. Сегодня была физика, практика, как и ожидалось решения более менее понятны до того момента, когда мы начинаем интегрировать. Жаль еще то, что я второй день не могу выспаться, соседи зажигают, а мне еще с недосыпом математику проходить.
>> No.145104 Reply
>>145097
Лучше спрашивай своих преподавателей, Сырно.
>> No.145110 Reply
>>145103
> сегодня я решаю огромное кол-во матриц и матричных уравнений, но что поделать, надо решить, чтобы понять как я усвоил материал
Но это же совершенно рутинные вещи. Зачем их решать?
> решения более менее понятны до того момента, когда мы начинаем интегрировать
Если знаешь инглиш, можешь посмотреть https://www.khanacademy.org/math/integral-calculus.
А вообще, насколько хорошо ты понимаешь дифференцирование и пределы?
>> No.145126 Reply
File: Snap-2015-09-17-at-20.23.34.png
Png, 69.27 KB, 1085×641 - Click the image to expand
edit Find source with google Find source with iqdb
Snap-2015-09-17-at-20.23.34.png
Доброаноны, помогите пожалуйста, я не понимаю решение интеграла, объясните пожалуйста почему во второй строке можно переписать (sinx)^7 так как там переписывается?
>> No.145132 Reply
>>145126
Это дурацкое дрочево. Представляешь седьмую степень синуса как куб квадрата синуса умноженного на ещё один синус. Квадрат синуса это 1 минус квадрат косинуса, возводишь в куб, умножаешь на оставшийся синус и вуаля...получаешь неудобоваримую ерунду.
>> No.145134 Reply
>>145126
Подозреваю, что используется какая-нибудь идиотская никому не известная тригонометрическая формула для задротов. Решать так - извращение. Воспользуемся тем, что cos^2(x) = 1-sin^2(x) и перепишем интеграл как sin^7(x) (1-sin^2(x))^3 cos(x) dx. Введем переменную t = sin(x). Получим интеграл t^7(1-t^2)^2 dt, который считается просто раскрытием скобок. После обратной подстановки синуса вместо t получим идентичный ответ.
>> No.145135 Reply
>>145134
> t^7(1-t^2)^2 dt
   t^7(1-t^2)^3 dt, конечно.
>> No.145136 Reply
>>145135>>145134
Тут так и решают вообще-то.
>> No.145145 Reply
>>144684
Аноны, помогите идиоту.
Репетитор посадил ботать логарифмы, я потихоньку и насобачиваюсь.

Но тут наткнулся на тип задач, которые понять никак не могу:
Нужно решить ур-ие наподобие следующих
7^(2x) + 7^x - 12 = 0
25^x + 2*5^x - 15 = 0

Выражать в итоге надо через логарифмы, а вот как подступиться, не понимаю.
>> No.145146 Reply
>>145145
А, отбой. Покопался в ГДЗ по другому учебнику, общий принцип понял.
До замены дошел и своим умом, но дальше по невнимательности не смог раскрутить.
>> No.145150 Reply
File: math.pdf
Pdf, 0.03 KB, 595×842 - Click the image to get file
math.pdf
>>144684
Все-таки застрял на логарифмах.
Нутром чую, что решаемо, аналогичный пример таки решил (хотя и пропустил очевидный без логарифмирования корень).
>> No.145152 Reply
>>145150
Подели на b^2, введи новую переменную t = a/b. Получишь квадратное уравнение относительно t. Реши его. Далее будешь иметь a = tb и из полученных значений t получишь уравнения на а. Однородных уравнений не знаешь, что ли?
>> No.145190 Reply
>>145110
> Но это же совершенно рутинные вещи. Зачем их решать?
Именно! Дело в том, что в моем институте препод по математике бабка лет 70, которая требует именно рутинные вещи.
Я и сам понимаю, что умнее от решения матриц не стану, но в качестве закрепления материала решаю их, хотя и без особого энтузиазма.
> А вообще, насколько хорошо ты понимаешь дифференцирование и пределы?
Примерно на уровне баки, т.е. дифференцирование у меня как-то выходит, хотя мне очень непонятно зачем везде ставить буквы d и т.п. если это просто производная, ну, значок дифференцирования есть значок, поэтому ставлю его и беру обычную производную, с интегрированием все сложнее у меня, хотя, вроде, это обратные вещи.
>> No.145206 Reply
File: main-qimg-76bc14b...
Png, 14.62 KB, 387×312
edit Find source with google Find source with iqdb
main-qimg-76bc14bf6bb1ba38b26ea4eac4a30744.png
File: 2000px-Euler_meth...
Png, 73.20 KB, 2000×1563
edit Find source with google Find source with iqdb
2000px-Euler_method.svg.png
File: linapprox.png
Png, 42.04 KB, 867×520
edit Find source with google Find source with iqdb
linapprox.png
File: Mvt2-2.png
Png, 84.50 KB, 748×684
edit Find source with google Find source with iqdb
Mvt2-2.png

>>145190
> хотя мне очень непонятно зачем везде ставить буквы d и т.п. если это просто производная
Отношение малых величин же. Смотри, пусть есть функция f, которую мы хотим линейно приблизить в x₀. Это будет выглядеть как-то так:
f(x₀ + Δx) = f(x₀) + A * Δx + o(Δx), где Δx — отклонение от точки x₀, o(Δx) — величина, которая много меньше Δx (при малых Δх).

Но это можно записать и по другому:
Δf = A * Δx + o(Δx)

Выразим A:
A = Δf/Δx + o(Δx)/Δx

Чем меньше Δx, тем ближе прямая и функция (пикрелейтед). В пределе же:
A = df/dx.

Понятно теперь, почему d?

Кратко об интегралах. Пусть у нас есть функция f, приближенная ломаной (второй пикрелейтед). Причём сетка равномерная, с шагом Δx.
Мы знаем, что наклон приближения на участке [x[n], x[n] + Δx] равен производной f'(c) в некой точке c внутри участка (есть отдельная теорема, но держи наглядный пикрелетед).

Пусть же теперь у нас нет ни функции f, ни её приближения, но есть сетка и наклоны, а кроме того, значение в начальной точке. Можем ли мы восстановить приближение?
Конечно можем: первый участок равен
L[0](x) = L(x[0]) + A[0] * (x - x[0])

а L(x[1]) находится как
L(x[1]) = L(x[0]) + A[0] * Δx

Остальные участки находятся аналогично. Да, на этот счёт тоже есть пикрелейтед.

Теперь пусть у нас нет ни сетки, ни наклонов, но есть f'. Не беда! Если функция гладкая, то наклон в малых окрестностях меняется мало, а потому при малых Δx мы можем взять любые с[n] на участках [x[n], x[n] + Δx], посчитать A[n] = f'(c[n]) и построить сносное приближение.

Пусть у нас нет начальной точки, но надо посчитать разность f(b) - f(a). Из сказанного выше видно, что она примерно равна
f(b) - f(a) ≈ L(b) - L(a) = f'(c[0]) * Δx + f'(c[1]) * Δx + ...

Или, если короче:
Σ f' * Δx = f(b) - f(a)

Внимание! Производится ПРЕДЕЛЬНЫЙ ПЕРЕХОД:

∫ f'(x) dx = f(b) - f(a) да, ∫ от enwiki://Long_s произошло

Поздравляем! У вас ИНТЕГРАЛ РИМАНА.

PS: нет, равномерная сетка вовсе не обязательна, просто наглядна
>> No.145227 Reply
>>145206
> Понятно теперь, почему d?
Ну, вроде бы понятно, мы рассматриваем отношение приращения функции к приращению ее аргумента, чтобы это сделать как можно точнее мы используем как можно более меньшие промежутки Δx и смотрим на приращение функции, так ведь? Т.е. такой глупые вопрос, буква d это и есть такое дельта, что изменение функции стремится к 0? т.е. это просто маленькое приращение, так ведь? И вместо стандартного значка делать мы заменяем его на более точный d?
> ∫ f'(x) dx = f(b) - f(a)
Мы же получили определенный интеграл, правильно же? т.е. от а до b. А как быть с неопределенным интегралом? разве площадь под графиком не может получиться бесконечной?
Вообще огромное спасибо тебе, доброняша!
>> No.145234 Reply
>>145227
> Ну, вроде бы понятно, мы рассматриваем отношение приращения функции к приращению ее аргумента, чтобы это сделать как можно точнее мы используем как можно более меньшие промежутки Δx и смотрим на приращение функции, так ведь?
Типа того. Эта запись ещё из тех времён, когда бесконечно малые считались чем-то реальным. Но общая суть всё та же: мы смотрим на отношение приращение функции к приращению её аргумента, но в пределе, где приращение аргумента стремится к нулю. С расчётом на то, что в пределе приближённые равенства переходят в точные.
К слову, физики принято говорить о dx как о малых приращениях/частях прямой, плоскости или пространства. У математиков, впрочем, не принято.
> Мы же получили определенный интеграл, правильно же? т.е. от а до b.
Он самый, хотя лучше его называть интегралом Римана.
ruwiki://Интеграл_Римана
> А как быть с неопределенным интегралом?
А вот это всего лишь абстрактная операция, которая сопоставляет функции f множество её первообразных Ф + C. Связывает их основная теорема анализа, согласно которой, если у нас есть первообразная Ф, то можно вычислить интеграл Римана как
∫_{a}^{b} f(x) dx = Ф(b) - Ф(a).

Но ведь можно написать и по другому. Пусть мы хотим найти какую-нибудь первообразную. Зафиксируем какое-нибудь a, и тогда:
Ф(b) = ∫_{a}^{b} f(x) dx + Ф(a)

Или, в несколько других обозначениях:
Ф(x) = ∫_{a}^{x} f(x) dx + C здесь надо различать то x, что задаёт отрезок интегрирования, и то x, что под интегралом
>> No.145242 Reply
>>145234
> Эта запись ещё из тех времён, когда бесконечно малые считались чем-то реальным.
Сейчас все сильно поменялось? Т.е. то, что я прохожу безнадежно устарело?
> С расчётом на то, что в пределе приближённые равенства переходят в точные.
Т.е. наш ответ получается каким-то приближение, которое может быть как-то пропорционально размеру Δx? Т.е. чем меньшее приращение аргумента берем, тем более точные у нас вычисления? Заранее извиняюсь, если написал бред.
> сопоставляет функции f множество её первообразных Ф + C
Я анализа пока не проходил, на математике сейчас глупые матрицы, но я не совсем понимаю зачем нам это сопоставлять? Я знаю, что множество С - бесконечно, часом не отсюда у нас получается неопределенный интеграл? Хотя перечитав еще раз пост, вроде как у нас есть бесконечное число Ф + С функций, теперь мы можем работать с этим множеством, скажем посчитаем не Ф + С, а Ф(b)-Ф(a)(мы взяли определенные функции из всего множества С) т.е. неопределенный интеграл это как бесконечная продуктовая лавка, где можно выбрать товары с 59 по 41 и попробовать, извиняюсь за глупую аналогию.
Ну, вроде как понятно, хотя правильно ли я все понял?
>> No.145247 Reply
File: question.PNG
Png, 22.11 KB, 1104×279 - Click the image to expand
edit Find source with google Find source with iqdb
question.PNG
>>144684
Аноняши, это снова шлимазл со школьной математикой.
Не могу понять задание на пикрилейтед.
Первую часть доказываю нормально, со вторым примером непонятки.

Это в учебнике ошибка (ответа нету, увы) или модули как-то влияют на результаты.
Просто сомневаюсь чтобы было возможно что |a/b|=|a||b|.
>> No.145248 Reply
>>145247
Ошибка в книге. В 2) должна быть разность логарифмов.
>> No.145255 Reply
>>145242
> Сейчас все сильно поменялось? Т.е. то, что я прохожу безнадежно устарело?
Нет. Просто нужно различать запись и её значение. Запись оказалась удобной, и потому сохранилась, её трактовка же полагается на понятие предела.
> Я анализа пока не проходил, на математике сейчас глупые матрицы, но я не совсем понимаю зачем нам это сопоставлять?
Смотри, пусть у нас есть функция f(x) = 2x. Функция Ф(x) = x^2 будет её первообразной. Но также её первообразной будут функции Ф(x) = x^2 + 1 или Ф(x) = x^2 + п или даже Ф(x) = x^2 - 1/e. Любая константа при дифференцировании даст нуль, а потому у функции f есть целое множество первообразных, отличающихся на константу.
Зачем нам нужны первообразные? Они позволяют аналитически находить интеграл Римана как разность Ф(b) - Ф(a), где Ф, очевидно, может быть любой первообразной f.
Зачем нам нужен интеграл Римана? Для расчёта множества вещей. Если мы можем что-то посчитать как k*Δx , когда k постоянно, мы можем посчитать это как ∫_{a}^{b} k(x) dx, когда k зависит от x. Скажем, таким k может быть скорость, а вместо x — время t: интеграл при этом равен пройденному пути от момента t₁ до момента t₂.
>> No.145264 Reply
>>145255
Спасибо тебе, в принципе стало больше ясности, кстати, капча: "объяснил бой" говорит сама за себя. Сейчас решил посмотреть лекции хана, в принципе неплохое дополнение к моим уже имеющимся знаниям.
>> No.145314 Reply
>>145080
>>145206
Респект товарищу за хорошее объяснение основ матана. Хотелось бы дополнить.
Польза от значка d(это называется "нотация Лейбница" - если не ошибаюсь) в отличии от обозначения штриха производной отчасти заключается в инвариантности формы первого дифференциала, удобстве при решении дифференциальных уравнений, при решениях физических/прикладных задач. Попробую объяснить на пальцах.
Пример:
Определение. df/dt=f(t)'
Чем же лучше df/dt чем f(t)'
А вот чем: df/dx*dx/dt=df/dt(оперируем как с обычными дробями - получили правило дифференцирования сложной функции f(x(t))).
df/dx=1/dx/df(формула производной обратной функции(если она существует) - нам не важно в записи, f зависит от x или x от f)
Интегралы - там значок d самый что ни на есть дифференциал - что необходимо учитывать при подстановке переменной - Допустим мы хотим применить подстановку в интеграле(чтоб его взять т.е. вычислить). Нам нужно преобразовать ∫ f(x) dx, t=g(x) -> ∫ h(t) dt, т.е. надо сделать подстановку для дифференциала тоже, для этого используем dt=g'(x)dx - подстановка в интеграле не так тривиальна, как производная сложной функции(аналог подстановки), нужно представить f(x) dx=h(t)*(g'(x)dx)=h(t)dt. То же самое касается интегрирования по частям(формула-метод вычисления интеграла - следует из формулы производной произведения).
Физика.
Рассмотрение бесконечно малых элементов аналитически заданных кривых, поверхностей, объемов(все законы записаны для точечных источников полей/материальных точек - "в дифференциальной форме") - сводится к рассмотрению их дифференциалов и интегралов. Именно оттуда исторически пошёл сам "смысл" дифференциала. Применяется при вычислении момента инерции, полей - всё это нужно в механике. Даже второй закон Ньютона включает в себя вторую производную траектории, задаваемой в общем виде функцией s(t) - векторной для двумерного/трёхмерного случая.
Дифференциальные уравнения - следующая ступень. Вкратце - в них включены неизвестные функции и их производные в одной точке. Используя основные законы физики выводим уравнения для динамических систем и "приложений"(механика, гидродинамика, электродинамика). Там нотация Лейбница используется повсеместно.
Например, метод решения дифф. уравнений "разделением переменных" не мыслим без использования символов dy и dt. Мы можем оперировать дифференциалами dy и dt раздельно, а не только в виде производной dy/dt.
С нотацией y(t)' так не прокатит.
dy/dt=f(y)f(t)(правая часть представима в таком виде)
dy/f(y)=f(t)dt <-> ∫1/f(y)dy=∫f(t)dt <-> переход к первообразным(берем интегралы с обоих сторон) F(y)=G(x)+C решаем относительно y, радуемся.
Поезд следует далее до остановки векторный анализ, дифференциальные формы и так далее - об этих разделах имею только смутное представление(т.к. учился на погромизда, а не на физика).
Про интегралы - там тоже много каких "вариаций" есть - криволинейные, кратные и прочие, нужные в физике многомерные ужасы.
Капча опять жжот - "вспоминая переносила". Только я не тня лол.
>> No.145324 Reply
File: IMG_20150921_205318.jpg
Jpg, 3894.48 KB, 4208×3120 - Click the image to expand
edit Find source with google Find source with iqdb
IMG_20150921_205318.jpg
>>144684
Выручайте, няши. Не получается p2 выразить из pобщ и p1
>> No.145350 Reply
Матаны, спасайте: пусть T(a) - параллельный перенос на вектор a. R(o, φ) - поворот вокрут точки о на угол φ. Всякое преобразование плоскости мы можем представить композицией R(o, φ)T(a). Пусть даны два преобразованияб f и g. f = R(o, φ)T(a), g = R(o, χ)T(b); h = gf = R(o, χ)T(b)R(o, φ)T(a). В результате этого преобразования поворот некоторой точки составит ψ = φ + χ, а результирующий вектор параллельного переноса окажется c = a + R(o, -φ)b. К сумме углов поворота вопросов нет. Это совершенно очевидно. Но откуда взялся этот поворот на -φ, почему -φ? Как так получается вообще? Не понимаю даже, с какого конца подойти. С "геометрической интуицией" всё совсем хуёво
>> No.145351 Reply
>>145324
Уже сам сделал, спасибо.
>> No.145358 Reply
>>145350
Ну что ж, посчитаем в операторах. То, что любое движение плоскости есть композиция поворота и сдвига на вектор а означает, что движение плоскости сопоставляет вектору x вектор А(х+а), где А - оператор поворота. Пусть А и А' - два оператора поворота, а и b - два вектора. Тогда композиция соответствующих движений будет переводить вектор х в вектор А'(A(x+a) + b) = A'Ax + A'Aa + A' b = A'A(x + a + (A)^(-1) b). Соответственно, композиция движений задаётся оператором AA'(поворот на сумму углов) и вектором a + (A)^(-1) b(из (A)^(-1) и берётся -ф).
>> No.145392 Reply
Что почитать по мат. анализу после школьной программы?
>> No.145393 Reply
File: tumblr_nowgqs2mds1rl1jado1_540.jpg
Jpg, 75.12 KB, 540×439 - Click the image to expand
edit Find source with google Find source with iqdb
tumblr_nowgqs2mds1rl1jado1_540.jpg
>>145392
Ты физик, математик, экономист или гуманитарий?
>> No.145394 Reply
>>145393
Поступил на физфак, но математику люблю больше. Пока что просто хочется разобрать основы понять, что такое пределы и т. д.
>> No.145395 Reply
File: B8JWS6nIAAEw0pr.jpg_large.jpg
Jpg, 49.20 KB, 604×604 - Click the image to expand
edit Find source with google Find source with iqdb
B8JWS6nIAAEw0pr.jpg_large.jpg
>>145394
Тогда тебе в принципе не нужна теория\теоремы\доказательства. Берешь задачник (Демидович подойдет) и пробуешь заниматься тем и этим. Решебник держишь по рукой. Вот есть не очень элитный сайтик, но полезный точно. http://mathprofi.ru/
Там можешь набраться инфы необходимой, чтобы начать. Но там расписано все с точки зрения практики, имей ввиду. Думаю после месяца не потных занятий основы знать будешь. Книг не знаю. Может какой другой анон подскажет. Хотя по опыту могу сказать, что не нужны, если физик.
>> No.145396 Reply
>>145392
Курант. "Что такое математика?"
Как минимум, прочитаешь про множества и про проективную геометрию.
>> No.145398 Reply
Привет, /u/. Долбоёб врывается итт.
Вопрос скорее по физике, чем по математике, но всё же.
Внутренняя энергия газа равна i/2vR*T . i, количество степеней свободы, равно три. Но почему так, если R равномощно R^3 ?
>> No.145399 Reply
>>145395
Cпасибо за сайт, теперь будет легче.
>>145396
Читал когда-то. Полезно, но там ведь не только мат. анализ. А мне нужно конкретную область побыстрее подтянуть. Но всё равно спасибо, достану эту книжку с полки и попробую выделить, что поважнее.
>> No.145404 Reply
>>144958
Бамп. Уже решил сам, теперь просто интересно сможет ли тут кто-нибудь другой.
>> No.145405 Reply
>>145398
Существуют понятия "конфигурационное пространство" и "фазовое пространство". Степени свободы - это из-за них. Ознакомься.
>> No.145564 Reply
File: Snap-2015-09-27-at-22.27.00.png
Png, 129.81 KB, 742×662 - Click the image to expand
edit Find source with google Find source with iqdb
Snap-2015-09-27-at-22.27.00.png
>>144684
доброаноны помогите
есть решение задачи, но не могу понять, как получается общее уравнение касательной к кривой, то есть как оно вообще составляется? объясните пожалуйста
>> No.145565 Reply
>>145564
См. >>145206.
Кратко: y' соответствует наклону кривой, (x - t) смещает её на t вправо, а прибавление y(x) — поднимает на соответствующую величину.
>> No.145568 Reply
>>145564
Халява, сэр. Уравнение прямой y=kx+b. Если прямая проходит через (x0, y0) то верно y0=kx0+b =>
y-y0=k(x-x0), k=f'(x) по определению производной. Ч.т.д.
>> No.145569 Reply
>>145404
Я пытался решить - на полпути лень/непонятно стало чёт. Попробую еще раз может.
>> No.145614 Reply
File: Снимок.PNG
Png, 2.74 KB, 220×142 - Click the image to expand
edit Find source with google Find source with iqdb
Снимок.PNG
Интересно, а как же всё-таки это решается? Определитель этой матрицы равен нулю, найти нужно x. П здесь - число пи, e - число Эйлера.
Анон, подкинь мне идею?
>> No.145619 Reply
>>145614
Раскрываешь определитель, приравниваешь нулю, решаешь уравнение. Это самый логичный способ, но не самый простой.
>> No.145634 Reply
>>145614
Ура, кажется решил. Для начала нужно заметить, что матрица симметрична и состоит из блоков(если провести линии по средней вертикали и горизонтали получим блочную матрицу
(A,B)
(B,A)
Искал - есть ли формула определителя для симметричных блочных матриц - подходящей не оказалось.
Еще тут есть связь с собственными значениями - тебя просят найти собственные значения матрицы, где вместо x стоят нули, собственное значение = -x. Определитель той матрицы с нулями = произведению корней характеристического уравнения. Но это всё были ложные ходы.
В итоге, пришел к тому, что надо использовать теорему Лапласа(разложить матрицу на сумму произведений миноров и их алгебраических дополнений по определенным строкам). Лучше всего выбирать строки 1,4(заметь в них симметрию 1 строка и столбец являются зеркальными отражениями) - там получаются определители одинаковых матриц - с точностью до перестановки строк и столбцов. Остаётся всё аккуратно посчитать(учесть знаки, привести одинаковые определители с помощью перестановок строк/столбцов, потом раскрыть их) - и мы у цели!
>> No.145635 Reply
>>145634
фикс
> (заметь в них симметрию 1 строка и столбец являются зеркальными отражениями)
на (заметь в них симметрию 1 строка и столбец являются зеркальными отражениями последней строки и столбца соответственно)
>> No.145638 Reply
File: IO8iYWmODBU.jpg
Jpg, 36.36 KB, 604×339 - Click the image to expand
edit Find source with google Find source with iqdb
IO8iYWmODBU.jpg
Моё почтение. На связи школьник-кун, гуманитарий. В этом году сдавать экзамены, а я с 5 класса хинал пуи на математике, а сейчас меня охватила паника. Боюсь что вообще не сдам математику, что делать? Как подготовиться за год?
>> No.145672 Reply
File: tumblr_mmred6jDXQ1s199fdo1_400.gif
Gif, 327.72 KB, 336×180 - Click the image to expand
edit Find source with google Find source with iqdb
tumblr_mmred6jDXQ1s199fdo1_400.gif
Аноны, у кого-нибудь есть таблица оригиналов и изображений в виде картинки нормально оформленной. В гугле какие-то фаршмачные. Мне просто нать с функцией ошибок\бессель и прочее. Если ни у кого нету, пойду сам клепать картинку...
>> No.145673 Reply
File: 1230601248658.png
Png, 617.94 KB, 2000×2000 - Click the image to expand
edit Find source with google Find source with iqdb
1230601248658.png
>>145206
>>145314
Спасибо вам еще раз за хорошее объяснение, решил отписаться и сообщиться, что савельев у меня идет вполне себе хорошо(механика) и я скорее понимаю смысл написанного. Кстати, как я понимаю дифференцирование и интегрирование - это анализ бесконечно малых величин, да?(все же я не очень догадливый и осенило меня только недавно, хотя ответ лежал под носом). Это ведь получается очень мощный у нас инструмент для познания функций, изменяющихся со временем! И вот у меня есть еще вопрос, можно ли сказать, что в очень малых приближениях функция ведет себя как производная? Или я немного не в ту сторону пошел?
>>145638
http://mathus.ru/
Попробуй его, он вполне себе хорошо объясняет, ну и тестики порешай, чтобы знать в каких заданиях спотыкаешься.
>> No.145677 Reply
File: wY0ayUk.jpg
Jpg, 2225.05 KB, 3200×2368 - Click the image to expand
edit Find source with google Find source with iqdb
wY0ayUk.jpg
>>145638
Академию Хана посмотри, если совсем элементарных вещей не знаешь.
>> No.145679 Reply
>>145672
Корн. Справочник по математике для научных работников и инженеров. Оттуда можно взять.
>> No.145686 Reply
>>145673
Дифференцирование и интегрирование - существенная часть "анализа бесконечно малых величин". Основа - теория пределов. Вообще исторически получилось, что понятие производной и интеграла появилось раньше понятия предела(и было недостаточно строгим). А сейчас они вводятся через предел т.к. это позволяет избежать интуитивно правильных, но формально неверных суждений. Можешь почитать на википедии(к примеру) про историю дифференциального и интегрального исчислений - довольно интересно и поучительно.
> Это ведь получается очень мощный у нас инструмент для познания функций, изменяющихся со временем!
Верно - производные и интегралы очень полезны и сами по себе, и для анализа поведения интегрируемых/гладких функций.
> можно ли сказать, что в очень малых приближениях функция ведет себя как производная
Можно. Почитай про приблизительные вычисления значений функций с помощью дифференциалов(используем приближение deltay~f'(x0)*deltax в окрестности точки x0), есть более общий случай(с производными высшего порядка) - формула Тейлора.
>> No.145688 Reply
File: Monogatari-(Series)-Anime-Anime-Art-MC4-2461080.jpeg
Jpeg, 130.99 KB, 640×640 - Click the image to expand
edit Find source with google Find source with iqdb
Monogatari-(Series)-Anime-Anime-Art-MC4-2461080.jpeg
>>145679
Ок, пасиба, он у меня как раз был на харде.
>> No.145711 Reply
>>145569
Норм, задача почти бесполезная, если слишком лень -- смысла делать не имеет. Сам бы не трогал, если бы домашкой не было.

На этой ноте реквестирую аспирантов/старшекурсников-математиков для ответа на вопрос: нужно ли хорошо шарить в планиметрии/стереометрии, если хочешь околопрофессионально заниматься геометриями всякими? Стоит ли прокачаться до уровня всеросса?
>> No.145712 Reply
>>145711
> колопрофессионально заниматься геометриями всякими
Что ты имеешь в виду? Алгебраическую геометрию, дифференциальную, алгебраическую топологию?
Впрочеме, в любом случае ответ "скорее нет". Школьный курс с современной математикой не пересекается.
>> No.145713 Reply
>>145712
Ну, алгем, диффгем, всякие оттенки топологии и т.п., да. Точнее пока не ясно совсем. В идеале бы ты ещё написал о своей квалификации, анончик. Всё таки важно не забить хуй на что-то, что потом может оказаться оч полезным.
>> No.145715 Reply
>>145713
Алгебраическая геометрия, теория особенностей, аспирант. В школе был олимпиадником, пару раз ездил на всерос. Знание олимпиадной математики мне в университете не помогло НИ РАЗУ и НИ В ЧЕМ, и я не преуменьшаю. Помогли некоторые прочитанные до университета хорошие книги.
>> No.145717 Reply
>>145715
Прекрасно! Очень благодарен за ответы.
>> No.145719 Reply
>>145686
Спасибо тебе большое, постараюсь посмотреть в ближайшем времени формулу Тейлора. Вообще мне пока нравится анализ, читать решил не сайтики, а Зельдовича как тут советовали выше, он очень доступно объясняет, прочитал раздел производных, посмотрел вообще как выводятся формулы дифференцирования и добрался до раздела интегрирование, в общем спасибо тому доброанону, что скидывал вверху эту книжку и спасибо за ответы!
>> No.145728 Reply
File: 69f1ad507b7f726f6682505b628f72af.jpg
Jpg, 121.56 KB, 842×870 - Click the image to expand
edit Find source with google Find source with iqdb
69f1ad507b7f726f6682505b628f72af.jpg
>>145719
Какой конкретно Зельдович. Уточните, пожалуйста.
>> No.145737 Reply
>>145728
Для начинающих физиков и математиков.
>> No.145738 Reply
File: 0a0a9873ecbf353fc94f5048e24f846f.png
Png, 979.04 KB, 1000×1000 - Click the image to expand
edit Find source with google Find source with iqdb
0a0a9873ecbf353fc94f5048e24f846f.png
>>145737
Спасибо.
>> No.145744 Reply
Аноны, чет я туплю оче сильно.
Смотри, у нас есть R^n
в нем есть операторы: переноса, растяжения и поворота
Вот вопрос, применение этих операторов коммутативно же? Ну геометрически нам вроде пофигу сначала перенести фигуру и потом крутануть/растянуть ее или же сначала растянуть/крутануть фигуру а потом перенести.
У меня в вузике семинарист сказал что имеет значение, но блин...
Крч можешь ли доказать/опровергнуть коммутативность этих операторов? (вообще была идея взять n-мерный симплекс для доказательства и покрутить его, но чет там по моему слишком дохрена вычислений, а в литературе я такого вопроса не встречал)
>> No.145749 Reply
File: 1.gif
Gif, 3179.54 KB, 377×500 - Click the image to expand
edit Find source with google Find source with iqdb
1.gif
Коши гад.
>> No.145751 Reply
>>145744
все норм, они не переносили подпространство относительно которого вращают/растягивают
>> No.145803 Reply
>>145744
Бёрешь такой точку на плоскости и крутишь вокруг неё - она остаётся на месте - а потом сдвигаешь. А теперь сначала сдвинь, а потом крутни (вращение относительно старой точки).
>> No.145811 Reply
>>145744
Вся фишка некоммутативности поворотов в пространстве - в том, что крутить можно вокруг разных осей. Если только вращать вокруг одной и той же оси - то можно переставить операции. Но уже добавляя перенос, коммутативность нарушается(меняется расстояние от объекта до оси - дуга поворота будет другой). Некоммутативность поворотов показывается простым геометрическим примером с поворотами вокруг разных осей(конкретно видел пример с поворотами куба на pi/2 вокруг координатных осей).
Определить свойства афинных преобразований можно такими способами:
Рассматривая матрицы преобразований в однородных координатах(использование однородных координат позволяет записать параллельный перенос и перспективное преобразование в матрицах). В общем случае умножение матриц ассоциативно, но не коммутативно. Для конкретных видов матриц(т.е. определенных преобразований) определяются их конкретные свойства.
Рассматривая кватернионы для поворотов/масштабирования - правда, кватернионы могут описывать только поворот/растяжение, если не ошибаюсь.
>> No.145825 Reply
File: Snap-2015-10-05-at-23.22.11.png
Png, 26.69 KB, 665×289 - Click the image to expand
edit Find source with google Find source with iqdb
Snap-2015-10-05-at-23.22.11.png
доброаноны, мне нужна помощь
я не понимаю решение интеграла, почему при замене t/2=u за знак интеграла выносится 4 и выражение становится 32, если du=(tdt)/2, и за знак интеграла вроде бы должна выноситься двойка?
>> No.145831 Reply
>>145825
Там ещё и пределы интегрирования уменьшили не вдвое, а вчетверо. Зачем-то воспользовались симметричностью синуса в точке п/2.
>> No.145842 Reply
File: 17b83a5cf688da3732102a58dba42c01.gif
Gif, 950.21 KB, 500×459 - Click the image to expand
edit Find source with google Find source with iqdb
17b83a5cf688da3732102a58dba42c01.gif
>>144684
Доброматематики, вопрос, полагаю, уже десять раз обсосанный, но буду очень признателен, если кто-нибудь посоветует добротного автора для изучения школьного курса этой самой математики.
В своё время доджил оную дисциплину как только мог, а теперь вот пришла нужда.
>> No.145846 Reply
>>145842
Мордкович - 5,6 классы, математика.
Макарычев - 7,8,9 классы, алгебра.
Колмогоров - 10-11 классы, алгебра.
Смирнова, Смирнов - 7-9 и 10-11 классы, геометрия.
Разумеется, бери углубленный уровень.
http://www.alleng.ru/edu/math1.htm
>> No.145847 Reply
>>145846
Огромное тебе спасибо.
>> No.145879 Reply
Что делать если я тупой, а учиться хочу хорошо? Линал, например, идет довольно туго, хотя я все определения нужные знаю, наверное...

Назовем две тройки двумерных подпространств в R^3 эквивалентными, если одну можно перевести в другую линейным автоморфизмом R^3. Сколько существует классов эквивалентности троек? Тот же вопрос для четверок двумерных подпространств в R^3.
>> No.145886 Reply
>>145879
Двумерное подпространство в R^3 есть плоскость. Плоскость однозначно задаётся одним вектором - вектором нормали к ней. Три подпространства - три нормали. Если эти три нормали образуют базис в R^3, то эта тройка подпространств эквивалентна любой другой тройке подпространств, нормали к которым образуют базис, поскольку любой базис переводится в любой другой автоморфизмом. Понятно также, что мы не можем подействовать на линейно независимую тройку автоморфизмом и получить тройку линейно зависимую. Поэтому все тройки подпространств, нормали к которым линейно независимы, эквивалентны. Теперь пусть нормали порождают не всё R^3, а подпространство размерности два, т.е. третья нормаль есть линейная комбинация первых двух с некими коэффициентами. Подействовав на тройку автоморфизмом, получим опять тройку вектором, в которой два вектора линейно независимы, а третий есть линейная комбинация первых с теми же коэффициентами, что и в предыдущем предложении. Значит, эквивалентность получается только при совпадении коэффициентов, что даёт нам континуум классов эквивалентности. Тройки нормалей, задающей пространство размерности 1, не бывает - это означало бы, что все три подпространства совпадают. Про четвёрки думай сам - там те же рассуждения, с поправкой на то, что четвёрка векторов в R^3 всегда линейно зависима.
>> No.145887 Reply
>>145886
Спасибо, но всё же, как такие задачи решать? Нужно как-то геометрически себе всё представлять, или иметь iq выше комнатной температуры?.. Больше практики нужно?
>> No.145888 Reply
>>145887
Единого рецепта нет и быть не может. Представлять геометрически не всегда полезно, зачастую наиболее естественный и простой путь - через алгебраические вычисления в координатах. Практики никогда не бывает много. Про iq сам всё понимаешь, думаю.
>> No.145942 Reply
Привет, я учусь на первом курсе околоматематической специальности. До этого математикой фактически никак не занимался.
Я более-менее нормально понимаю лекции и разборы задач на семинарах тоже более-менее понимаю, но проблема в том, что задачки решать у меня практически не получается. Дискретная математика еще куда ни шло, а вот алгебра и матанализ у меня вообще не идут.
Как это просходит: есть какое-то дз (например), я сажусь его решать. Обычно более-менее быстро решаю несколько простых задач (процентов 25% от дз), но дальше у меня не идет. Я понимаю условие задачи, но не могу ничего сделать. Могу сидеть два часа над одной (относительно не сложной) задачей, но никак не приближаюсь к решению. Иногда я думаю, что я, наверное, чего-то не понял, начинаю читать конспект/какой-нибудь учебник и убеждаюсь в том, что в общем-то все понял, но все равно не могу ничего решить.
Что ты мне посоветуешь? Просто больше решать, даже если КПД очень низкое, и скилл сам по себе прокачается?
>> No.145949 Reply
>>145942
Поддерживаю вопрос.
>> No.145950 Reply
File: [HorribleSubs]-Owarimonogatari-01-[720p].mkv_snaps.jpg
Jpg, 83.51 KB, 1280×720 - Click the image to expand
edit Find source with google Find source with iqdb
[HorribleSubs]-Owarimonogatari-01-[720p].mkv_snaps.jpg
>>145942
Очевидно нужен опыт. Только с ним выработаешь интуицию для решения задач. Придется немного позадротить.
>> No.145952 Reply
>>145942
Приведи пример задачи, ставящей тебя в тупик. А лучше нескольких.
Навскидку, ты просто неопытен и тебе нужно попрактиковаться. Возможно, у вас плохо ведутся семинары и преподаватель не может донести до вас методы решения задач. Возможно, дело в отсутствии привычки к длительной целенаправленной работе над задачей. Возможно, ты просто-напросто туповат. Без конкретных примеров можно только делать предположения.
>> No.145974 Reply
File: -.png
Png, 39.17 KB, 645×799 - Click the image to expand
edit Find source with google Find source with iqdb
-.png
Сап. Сталкнулся с матаном. До этого случая математики последний раз касался несколько лет назад. Объясните пожалуйста как это решается.
>> No.145988 Reply
>>145974
Тащемта, объяснять тут особо нечего, ибо это очень простые задания. Любые разъяснения будут, фактически, означать решение задачи за тебя. А так вся нужная инфа есть, например, тут:
ruwiki://Непрерывная_функция#.D0.9A.D0.BB.D0.B0.D1.81.D1.81.D0.B8.D1.8[...]C2.B9
>> No.146110 Reply
>>145711
Извини, что поднимаю проблему больше чем, через месяц - но как та >>144958 задача решается? Я дошел только до того, что точки M и N лежат на пересечении срединных перпендикуляров из противоположных сторон, еще что сумма углов в тр-ках CND и ANB, кроме углов при вершине N = 2pi/3. Аналогично, сумма углов в тр-ках DMA и CMB без углов при M = 2pi/3. Что делать дальше?
>> No.146212 Reply
>>144684
Аноны, мне нужна помощь. Помогите пожалуйста с задачкой:"Сколькими способами можно выбрать три различных числа из множества {1,2,...,n}, чтобы их сумма делилась на три?". Как решать такие задачи? Дискретка совсем не даётся((
>> No.146215 Reply
>>146212
Формула включений-исключений.
>> No.146220 Reply
>>146212
Кобминаторика, сэр.
Рассмотрим остатки от деления чисел. Чтобы сумма 3 чисел делилась на 3 - необходимо и достаточно, чтобы сумма их остатков была 0 или 3. В случае 0 - все 3 числа должны делиться на 3, таких чисел n/3, собираем из них сочетание без повторений по 3. В случае остатка=3 - остатки = 2,1,0, таких чисел(порядок чисел нам не важен) (n/3)(n/3+ 1) если (n mod 3>=1) иначе 0)(n/3+ 1 если (n mod 3==2) иначе 0), такие числа заведомо не равны. Суммируя C_(n/3)^3 и ранее полученное выражение - получаем результат.
>> No.146266 Reply
Что будет, если от бесконечности отнять единицу? Перестанет ли она быть бесконечностью, или превратиться в бесконечность-1?
>> No.146267 Reply
>>146266
Нет. Если отнять от бесконечности любое конечное число, ничего не произойдет. Также как к конечному числу прибавить ноль.
>> No.146273 Reply
>>144684
Привет, няши.
Я хочу ввести вероятностную меру "В широком смысле".

Суть:
Есть отрезок K = [0;1] в R.
P(K) = 1, P(K пересечение Q) = 0.5
P(A) = P(B) если у интервалов А и B равная длина. Существуют "рациональные" отрезки

Теперь я могу измерять
1. Вероятность попадания в отрезок
2. Вероятность попадания в "рациональный" отрезок

По дороге я потерял счётную аддитивность. Эт я знаю.

У меня подозрение, что где-то я приду к противоречию. Так это?
>> No.146317 Reply
Привет, доброчан. Подскажи хороших материалов для самостоятельного изучения векторной алгебры и тензорного анализа. Больше
>> No.146327 Reply
>>146273
> По дороге я потерял счётную аддитивность.
Ты имеешь в виду то, что определенная таким образом мера с необходимостью должна быть инвариантна относительно сдвигов на рациональные числа и возникает противоречие в том, что любое рациональное число имеет меру нуль, а их счетное объединение - положительную меру? Иначе я не вижу, где ты теряешь счётную аддитивность.
> У меня подозрение, что где-то я приду к противоречию.
К противоречию с чем именно? К противоречию с определением меры ты уже пришёл, о чём сам знаешь.

Я не очень понимаю, нафига тебе нужна такая мера.
> Теперь я могу измерять
> 1. Вероятность попадания в отрезок
> 2. Вероятность попадания в "рациональный" отрезок
И что это даёт?
>> No.146328 Reply
>>146273
>>146327
А, всё, сообразил, где у тебя противоречие.
> инвариантна относительно сдвигов на рациональные числа
Scratch that, она инвариантна относительно вообще любых сдвигов на действительные числа. Тогда противоречие простое: пусть [A, B] - отрезок длины 2. Сдвигая [0,1] \cap Q на достаточно маленькие иррациональные числа q1<q2<..<qn такие, что qi-qj - иррациональное для любых i<j, можно добиться того, что в множестве меры 2 будет содержаться множество меры n/2 для сколь угодно большого n.
>> No.146330 Reply
File: IMG_20151018_013344_1.jpg
Jpg, 777.19 KB, 2448×3264 - Click the image to expand
edit Find source with google Find source with iqdb
IMG_20151018_013344_1.jpg
>>144684
Решите пожалуйста, или объясните как решать.
>> No.146341 Reply
File: Снимок.PNG
Png, 1480.39 KB, 1268×724 - Click the image to expand
edit Find source with google Find source with iqdb
Снимок.PNG
>>146328
Тонны нефти!
>> No.146431 Reply
File: Snap-2015-10-22-at-20.36.53.png
Png, 58.92 KB, 836×169 - Click the image to expand
edit Find source with google Find source with iqdb
Snap-2015-10-22-at-20.36.53.png
>>144684
Объясните пожалуйста как решать этот номер, то есть как находить это линейное преобразование
>> No.146540 Reply
File: помогите2.jpg
Jpg, 58.99 KB, 606×1080
edit Find source with google Find source with iqdb
помогите2.jpg
File: помогите3.png
Png, 87.87 KB, 382×167
edit Find source with google Find source with iqdb
помогите3.png
File: помогите1.jpg
Jpg, 69.12 KB, 606×1080
edit Find source with google Find source with iqdb
помогите1.jpg

Доброаноны, помогите пожалуйста. В прошлом году не досдал геометрию, осталась всего одна задача.
Вот что у меня есть: решение (не факт, что правильное) одного из примерно 15 заданий этого типа, я попытался разобраться. То что я понимаю, написано синей пастой, вопросы золотой.
Пожалуйста помогите разобраться, завтра последняя пересдача. Не хочу, чтобы отчислили из-за одной задачи.
>> No.146541 Reply
File: 1.png
Png, 10.35 KB, 829×87
edit Find source with google Find source with iqdb
1.png
File: 2.png
Png, 17.41 KB, 854×156
edit Find source with google Find source with iqdb
2.png

>>144684
Анон, помоги понять решение задачи. Я чувствую, что не могу переложить условие на математический язык так, чтобы было удобно рассуждать, а задача сама по себе легкая. Вот условие:
> Рассмотрим элемент группы симметрий треугольника, которому при некотором обозначении вершин соответствовала подстановка h. Какая подстановка будет соответствовать этому же элементу группы симметрий треугольника при переобозначении вершин g?
Остальное смотри на скринах. Я запутался в этих композициях и порядке рассмотрения вершин (до и после преобразований). Разжуйте как для дауна, я не хочу переходить к следующим задачам, не разобравшись в этой.
>> No.146542 Reply
>>146540
Начнем с того, что ты неправильно списал условие задачи. Так что нет ничего удивительного в том, что у тебя получаются неприятные числа.
Сразу скажу, что я не помню, как эта задача решается "по учебнику", поэтому расскажу, как решал бы сам: квадратичная часть этой кривой имеет вид x(8x+6y). Из условия z+t = x, z-t = 8x+6y находим координаты z = 9/2x+3y, t=-3y-7/2x, в которых квадратичная часть имеет вид z^2-t^2. Добавляя линейную часть, получаем уравнение z^2-t^2+5/2z+3/2t+1=0. Дальше ничего не стоит сдвинуть начало координат. Канонические координаты сразу выписываются, исходя из хода решения.
>> No.146589 Reply
Аноны, как вы справляетесь со скукой математики? Мне в принципе интересна алгебра, теория категорий, логика. Хотелось бы разбираться, но как только начинаю читать книгу сразу идет некое отвращение от всех этих технических теорем и скучных доказательств. Раньше любил разбирать доказательства, а от красивого/неожиданного доказательства даже получал удовольствие. сейчас видимо развился толер, лол Да и сейчас как-то стал забывать формулировки теорем, т.е. если в тексте доказательства есть какой-то факт, мне надо сначала вспомнить, откуда он берется, но мне лень, в результате забиваю.

Короче, как снова вернуть интерес к математике?
>> No.146590 Reply
>>146589
Сдаётся мне, что это нетортянка.
Лечится так же как любая другая нетортянка.
>> No.146599 Reply
>>146590
Доктор, так что вы пропишете? Каков прогноз? Есть случаи выздоровления?
>> No.146633 Reply
>>146589
Попробуй повтыкать в совершенно новый/неизвестный/недолюбливаемый для тебя раздел, к примеру. Еще можно попробовать сделать перерывчик.
>> No.146644 Reply
File: shutterstock40849366.jpg
Jpg, 38.35 KB, 600×900 - Click the image to expand
edit Find source with google Find source with iqdb
shutterstock40849366.jpg
Привет, Математик-кун. Подскажи, пожалуйста, задачник по логике, типа "Практическая логика. Задачи и упражнения" Ивина?
>> No.146659 Reply
>>146644
Ну вот простой пример: предложение "Подскажи, пожалуйста" - повествовательное, в конце должна быть точка. А ты поставил вопросительный знак, значит, ты крайне нелогичное существо. Зачем тебе такие сложные учебники?
>> No.146661 Reply
>>146660
Где у тебя переменные, а где - функция? Мне не ясно.
> Существует ли не слишком хлопотный способ на основе наборов "значения переменных + значение функции" приближенно вывести формулу этой функции?
Ответ зависит от того, что ты подразумеваешь под "приближением". В любом случае, большинство известных мне способов интерполяции именно что хлопотны.
enwiki://Trilinear_interpolation
> Функция прямо пропорциональна всем переменным.
Если это действительно так, то она имеет вид f(x,y,z) = axy*z, a - константа. Эту константу ты можешь вычислить, попросту поделив значение функции на произведение аргументов для любого набора аргументов. Тем не менее, глядя на твою таблицу, я не наблюдаю там никакой прямой пропорциональности чего-либо чему-либо.
>> No.146748 Reply
File: 1412457375-74450.jpg
Jpg, 196.14 KB, 1000×1500 - Click the image to expand
edit Find source with google Find source with iqdb
1412457375-74450.jpg
>>145314
Вы так это все рассказываете, но некоторым все равно может быть непонятно о чем вообще идет речь.
Дифференцируемая функция в точке и производная в точке - два разных понятия, эквивалентность которых еще нужно доказать. Но они, как я заметил, сливаются у людей в одно, из-за чего вылезают порой глупые вопросы, мешающие понимать последующий материал. Наверное это из-за того, что некоторые авторы говорят об этих понятиях сразу же как об едином целом, опуская доказательство
>> No.146769 Reply
>>146748
> Дифференцируемая функция в точке и производная в точке - два разных понятия, эквивалентность которых еще нужно доказать.
Ты можешь мне объяснить смысл этой фразы? Либо я совершенно не понимаю твоей манеры приставлять слова друг к другу, либо ты говоришь что-то совершенно неверное.
>> No.146771 Reply
File: cV6ZJQQQt-8.jpg
Jpg, 131.62 KB, 960×719
edit Find source with google Find source with iqdb
cV6ZJQQQt-8.jpg
File: lzVXY1Z00WM.jpg
Jpg, 49.58 KB, 604×452
edit Find source with google Find source with iqdb
lzVXY1Z00WM.jpg

Что можно почитать или посомтреть, чтобы понять, как решать ЭТО?
Вузовский учебник какая-то ересь. Второй раз главы перечитываю и понятно ничего.
>> No.146775 Reply
File: 14266622077190.jpg
Jpg, 103.79 KB, 1200×800 - Click the image to expand
edit Find source with google Find source with iqdb
14266622077190.jpg
>>146769
И вот тут я встрял. Надо мне больше писать похоже
В общем, понятие дифференцируемости и производная сливаются у людей в одно, от чего возникает небольшое непонимание, мешающее дальше.
Хотя как сначала было-то. Вот мы рассматриваем производную, далее вводим дифференцируемость и потом доказываем, что этот коэффициент A в дифференциале и есть производная, и что эти понятия эквиваленты
dy и dy/dx некоторые (а ирл многие) считают одним и тем же, проще говоря.
>> No.146776 Reply
File: 14353521470470.jpg
Jpg, 184.29 KB, 960×720 - Click the image to expand
edit Find source with google Find source with iqdb
14353521470470.jpg
>>146771
Линейную алгебру Мальцева или книжку от мгту по линейной алгебре, если хочешь просто решать
>> No.146780 Reply
>>146776
Спасибо, книжка мгту помогла со всем, кроме 3.
>> No.146783 Reply
File: IMAG1722.jpg
Jpg, 492.14 KB, 2592×1552
edit Find source with google Find source with iqdb
IMAG1722.jpg
File: IMAG1721.jpg
Jpg, 540.21 KB, 1552×2592
edit Find source with google Find source with iqdb
IMAG1721.jpg
File: IMAG1720.jpg
Jpg, 592.46 KB, 1552×2592
edit Find source with google Find source with iqdb
IMAG1720.jpg

Доброчан, помоги с неравенством. Вот оно, и я его вроде бы как почти все решил, если бы только не проверка на 3-е ОДЗ. Вообще не знаю, что с ним делать, оно никак на множители не раскладывается и ничего с ним сделать незя. Что я делаю не так?
>> No.146788 Reply
File: _aa_src_77871_14306866726860.jpg
Jpg, 86.75 KB, 911×614 - Click the image to expand
edit Find source with google Find source with iqdb
_aa_src_77871_14306866726860.jpg
>>146780
Если эти скобочки угловые означают у вас линейную оболочку, то ищи пересечение линейных пространств. Оно и там и там хорошо описано
>> No.146791 Reply
>>146775
>>146748
> Дифференцируемая функция в точке и производная в точке - два разных понятия
Точнее - существование производной в точке. Из того, что помню из матана: дифференцируемость - это представимость приращения(deltay) в виде A*deltax+o(delta_x), что эквивалентно существованию производной=A в точке. Может быть, в других учебниках/в стародавние времена под дифференцируемостью понимали непосредственно существование производной(понятие предела и, соответственно, o малое появилось позже) - хотя могу ошибаться.
>> No.146793 Reply
>>146791
> Может быть, в других учебниках/в стародавние времена под дифференцируемостью понимали непосредственно существование производной
Я все ещё не понимаю, что ты несёшь. Ты сам сейчас под дифференцируемостью понимаешь непосредственно существование производной, коль скоро говоришь, что эти вещи эквивалентны.
>> No.146794 Reply
File: Безымянный.png
Png, 15.59 KB, 778×468 - Click the image to expand
edit Find source with google Find source with iqdb
Безымянный.png
Всем привет.
Чувствую, что немного отхожу от сути /u/, но надеюсь, что кто-нибудь поможет.

Пытаюсь вывести формулу неизвестной функции на основе множества точек. Могу увидеть округлённый результат функции. Вот только не могу свободно варьировать переменные. Вернее могу, но в жестко ограниченных небольших диапазонах для разных наборов переменных.

Попытался собрать немного данных и исследовать их на глаз.
Функция зависит от 4 переменных. Две переменные (x2 и x3) влияют на неё довольно хаотично, но их среднее значение при фиксировании других переменных даёт вот этот график. Очевидно, что они присутствуют в формуле именно в таком виде, хотя могут и по отдельности учитываться ещё в каком-нибудь виде (например, линейно).

График f(x1) мне очень сложно построить, поскольку остальные переменные очень сложно зафиксировать в сколь-нибудь значительном количестве точек.

f(x4) линейно возрастает.

Получаем что-то в духе:
f(x1, x2, x3, x4) = a0 + a1 ((x2+x3)/2)^2 + a2 * x4 + g(x1, x2, x3, x4)

Где g() - пока ещё не выведенная закономерность.
Про ^2 не совсем уверен.

Собственно, каким методом можно из всего этого вывести более менее точную формулу?

Когда думал, что все закономерности линейны, пробовал автоматически проводить линейную регрессию в MS Excel. Результат был не слишком плох, но когда добавил точки с низким значением переменных, модель, по очевидным причинам, оказалась бестолковой.
>> No.146795 Reply
>>146794
Независимое влияние x2 и x3 пока, тоже, трудновато вывести.
Вообще, по этой функции довольно хлопотно собирать данные.
>> No.146796 Reply
>>146794
> Две переменные (x2 и x3) влияют на неё довольно хаотично
Вру. Это я графики тогда неправильно построил (не все переменные зафиксировал).
>> No.146797 Reply
>>146794
> Две переменные (x2 и x3) влияют на неё довольно хаотично, но их среднее значение при фиксировании других переменных даёт вот этот график. Очевидно, что они присутствуют в формуле именно в таком виде, хотя могут и по отдельности учитываться ещё в каком-нибудь виде (например, линейно).
Не очевидно. Вполне возможно, что они присутствуют в виде названной тобой функции, домноженной на некоторую функцию, зависящую от остальных переменных. Второй множитель просто перестает влиять, когда ты фиксируешь остальные переменные.

Не надо изобретать велосипед. Скачай что-нибудь вроде wolfram mathematica и примени к своим данным какой-нибудь алгоритм интерполяции. Сможешь получить кучу приближенных формул и выбрать, какая понравится.
>> No.146798 Reply
>>146797
Лол, всё так очевидно.
Спасибо.
>> No.146831 Reply
File: Nora.Arendt.345432.jpg
Jpg, 90.04 KB, 500×668 - Click the image to expand
edit Find source with google Find source with iqdb
Nora.Arendt.345432.jpg
Привет. Хотел задать вопрос. Можно?
У нас есть множество А, в котрое входят разные города, и множество B, в которое входят разные страны. Когда мы сделаем декартово произведение AB, получим множество пар, состоящих из городов и стран. В этом множестве можно будет выделить подмножество R, в которое будут входить только те пары, для которых будет верно, что первый компонент пары будет город, являющийся столицей для страны, которая является вторым компонентом пары. При этом декартово произведение AB будет областью определения 2-местной функции f. А какое множество будет областью её значений?
>> No.146832 Reply
>>146793
Это уже другой анон
>> No.146833 Reply
File: _aa_src_77871_14266621274861.jpg
Jpg, 124.87 KB, 800×800 - Click the image to expand
edit Find source with google Find source with iqdb
_aa_src_77871_14266621274861.jpg
>>146793
>>146832
Пик отклеился
Теперь я не понимаю, что ты не понимаешь
>> No.146837 Reply
>>146831
О какой функции f идет речь? Для любого множества Х, мощность которого не больше мощности АВ, можно построить функцию с областью определения АВ и областью значений Х. Если ты имеешь в виду какую-то конкретную функцию, то назови её.
>> No.146839 Reply
>>146833
Если я хоть сколько-нибудь тебя понял, то ты говоришь следующее: существование производной в точке равносильно дифференцируемости в точке, но это не одно и то же. Так?
>> No.146846 Reply
File: _aa_src_77871_14271703514670.gif
Gif, 1298.27 KB, 639×463 - Click the image to expand
edit Find source with google Find source with iqdb
_aa_src_77871_14271703514670.gif
>>146839
3й раз буду писать одно и то же
Я говорю о том, что если сразу рассматривать эти понятия как эквивалентые, то у некоторых людей возникает мешающее непонимание: дифференциал они отождествляют с производной. Хотя должно быть
> существование производной в точке равносильно дифференцируемости в точке
Алсо, в своих постах я ничего не утверждал, а лишь рассказывал о случаях ирл
>> No.146850 Reply
>>146846

Начнем с того, что понятия не бывают эквивалентными. Эквивалентными бывают утверждения.
> если сразу рассматривать эти понятия как эквивалентые
А как еще можно их рассматривать как эквивалентные? Постепенно?
> у некоторых людей возникает мешающее непонимание: дифференциал они отождествляют с производной.
Это очень ценное наблюдение, но я не вижу какое отношение оно имеет к теме треда.
>> No.146852 Reply
>>146850
> понятия не бывают эквивалентными
Бывают. Понятие A эквивалентно понятию B, если понятие A есть необходимый и достаточный признак для B.
>> No.146867 Reply
>>146852
Т.е. у понятий бывают необходимые и достаточные признаки? Это новый для меня факт. Можешь назвать достаточный признак понятия "варенье"? А необходимый признак понятия "стул"? Я в разговоре с тобой просто совершаю открытие за открытием.
>> No.146883 Reply
>>146867
Чтобы объект был вареньем, достаточно, чтобы он был вареньем из вишни. То есть "варенье из вишни" - достаточный признак варенья. Чтобы объект был стулом, необходимо, чтобы он был мебелью. То есть "мебель" - необходимый признак стула.

Ну и да, я вмешался в ваш разговор начиная с поста >>146852, изначально ты с другим аноном говорил. Ваш разговор оказался настолько интересен, что я просто не смог пройти мимо. Ты что, правда не знал, что у понятий бывают необходимые и достаточные признаки?
>> No.146887 Reply
>>146883
> Ты что, правда не знал, что у понятий бывают необходимые и достаточные признаки?
Я до сих пор считаю, что их не существует. По очень простой причине: говорить о достаточных и необходимых признаках понятия неграмотно. Так не говорят. Можно говорить о необходимых и достаточных условиях истинности некоторого утверждения. Понятия не могут быть эквивалентными, потому что они не могут быть истинными и ложными.
В частности, твои контрпримеры относятся не к области понятий, а к области необходимых и достаточных условий истинности некоторых утверждений, а именно утверждений "этот предмет является стулом" и "этот предмет является вареньем". Моё возражение было этимологического, а не логического толка.
>> No.146889 Reply
>>146887
Разумеется, так говорят. Понятие есть следующий набор данных:
1. Некоторая строка символов, которая называется "термин"
2. Некоторый набор одноместных предикатов P1, P2, ... , Pn, который называется содержанием понятия.

С каждым понятием свяжем одноместный предикат P, истинный тогда и только тогда, когда истинен предикат P1∧P2∧ ... ∧Pn, и назовём его результирующим предикатом. Если P - результирующий предикат понятия, если x - какой-то объект, и если P(x) истинно, то мы будем называть x с помощью термина, связанного с понятием. А именно, мы будем говорить "x есть <термин>". Далее, класс (в смысле NBG) объектов, заданный как {x | P(x)}, мы будем называть объёмом понятия.

Пусть p и q - два понятия. Пусть P и Q их, соответственно, результирующие предикаты. Пусть истинно, что ∀x P(x)→Q(x). Тогда мы говорим, что понятие q есть необходимый признак понятия p, понятие p есть достаточный признак понятия q.

Строки символов, которые являются термином для какого-то понятия, мы будем называть осмысленными. Строки, которые не являются осмысленными, мы будем называть бессмысленными. Мы будем считать, что у двух разных понятий не может быть одного и того же термина; омонимия запрещена. Это позволяет нам установить взаимно-однозначное соответствие между термином понятия и самим понятием и при желании свободно их взаимозаменять, не опасаясь путаницы.

Этим и обосновано использованное мной словоупотребление. Ты понимаешь под понятием что-то другое?

предыдущий пост удолил из-за опечатки
>> No.146890 Reply
>>146889
> Строки символов, которые являются термином для какого-то понятия, мы будем называть осмысленными.
Это место мне неясно. Без ограничения на выбор строки в первом пункте твоего определения у тебя все строки окажутся осмысленными. Не легче ли считать, что понятие определяется своим результирующим предикатом? В конце концов, зачем делать название существенной частью математического объекта? По-моему, это как многообразие определять, как объект, состоящий из двух частей : 1) слова "многообразие"; 2) топологического пространства с набором карт.

Возражения принимаются, с учётом того, что я всё же говорил не с тобой, и человек, с которым я говорил изначально, имел в виду не это.
>> No.146891 Reply
>>146890
> Это позволяет нам установить взаимно-однозначное соответствие между термином понятия и самим понятием и при желании свободно их взаимозаменять, не опасаясь путаницы.
А вот и не позволяет. Для взаимной однозначности тебе также нужно поставить условие о том, что у разных понятий должен быть разный набор предикатов. Твоё текущее определение не учитывает возможности существования понятий с одинаковыми терминами, но с разными наборами предикатов.
>> No.146892 Reply
>>146891
> Для взаимной однозначности тебе также нужно поставить условие о том, что у разных понятий должен быть разный набор предикатов
Гм, третий час ночи, пора спать. Не об этом тебе нужно поставить условие, а о том, что у понятий с одинаковыми терминами должен быть одинаковый набор предикатов. Последнее предложение остается в силе.
>> No.146894 Reply
>>146892
Точнее, для взаимной однозначности нужно, чтобы у понятий с одинаковым содержанием был один и тот же термин. Согласен. Зря я там про взаимную однозначность ляпнул, ведь подразумевал инъекцию же. Специально хотел рассмотреть ситуацию, когда были бы возможны разные понятия с одинаковым содержанием.

>>146890
Осмысленными являются только те строки, для которых найдётся понятие, термином которого они являются. Например, если имеется понятие, термином которого является строка йцукен, то этот набор символов осмыслен, иначе бессмыслен. Описание понятий относится к метаязыку, поэтому вопрос об осмысленности той или иной строки относится к нему же (или даже к метаметаязыку).

В реальном мире возможна ситуация, когда два понятия имеют одно и то же содержание, но различаются термином. Поэтому для нужд эпистемологии всё-таки желательно считать термин важной частью понятия. Идея в том, что всякое понятие рассматривается как некий детектор, который любую вещь из предметной области может проверить на наличие заданного набора свойств и повесить на неё бирку с неким текстом в случае, когда эти свойства у вещи присутствуют. Причём свойства вещи называются в этом случае её аксиомами. Всякий набор аксиом при таком подходе оказывается определением какого-то объекта, и определение всякого объекта оказывается всего лишь перечислением его аксиом. В частности, многообразие - это набор аксиом, поименованный словом "многообразие". Давать наборам аксиом имена нужно, главным образом, для удобства. Ничто не мешает пользоваться наборами аксиом, которые не имеют термина.
>> No.146897 Reply
>>146889
Это враньё
> Разумеется, так говорят.
Остальное чепуха. Может, для философов сгодится, хотя... всё-таки нет.
>> No.146898 Reply
>>146897
Нет, это не враньё.
>> No.146899 Reply
>>146898
Я лично такой бессмыслицы не встречал. Сделай полнотекстовой поиск по книгами через гугл для большей выборки.
>> No.146900 Reply
>>146899
Это означает, что ты не читал учебники логики (обычной, не математической). "Признак понятия" - вполне респектабельное логическое выражение, встречающееся в книжках ещё с имперских времён. Например, см. >>145631, глава III.
>> No.146902 Reply
>>146900
Точнее, терминология, которая нигде не встречается кроме безумных русских книжек по неформальной логике.
Например, здесь говорят только о необходимости и достаточности условий http://plato.stanford.edu/entries/concepts/.
Да и в любом случае не важно, потому что речь о математике.
>> No.146905 Reply
>>146902
Тем не менее, так говорят, и этой терминологии даже может быть дан точный смысл. Нет ничего плохого в том, чтобы сказать, например, "квадрат с необходимостью есть параллелограмм" или "всякая селёдка с необходимостью есть рыба".
>> No.146911 Reply
>>146905
> Нет ничего плохого в том,
Есть плохое. "С необходимостью" относится к связке "есть" и не несет никакого смысла. В таком употреблении оно не имеет ничего общего с необходимостью/достаточность условий, которые указывают на косеквент и антецедент импликаций.
Можно сделать простой перевод: "x есть y" в "для любого a, (a удовлетворяет определению x) -> (a удовлетворяет определению y)". Языковые функции x как подлежащего и y как части сказуемого указывают, что посылка, а что следствие.
>> No.146912 Reply
>>146911
Я только что объяснил смысл. Общее есть. Ты своим переводом именно это общее вскрыл, и мне теперь непонятно, с чем ты споришь. Если ты считаешь, что утверждение "всякая селёдка с необходимостью есть рыба" можно понять неправильно, то покажи, как.
>> No.146914 Reply
>>146912
> можно понять неправильно, то покажи, как.
"Селедка есть рыба." "С необходимостью" лишнее и бессмысленное.
>> No.146915 Reply
>>146914
Нет, не лишнее. В естественном языке "есть", не имеющее уточнений, может означать необходимость и достаточность, поэтому утверждение "селёдка есть рыба" невозбранно можно понять как "селёдка - синоним для рыба". Во избежание недоразумений нужно уточнять, что селёдка есть рыба лишь с необходимостью, но не с достаточностью.
>> No.146923 Reply
>>146915
> > В таком употреблении оно не имеет ничего общего с необходимостью/достаточность условий, которые указывают на косеквент и антецедент импликаций.
>> No.146924 Reply
>> No.146928 Reply
>>146924
Это пустое. Дальше что?
>> No.146930 Reply
>>146928
Я жду аргументов. Общее я указал.
>> No.146935 Reply
>>146930
Аргументов почему пустое? Потому что это словоупотребление (необходимое/достаточное понятие) не встречается в нормальной литературе (определенно в математической). Конъюнкция предикатов, кстати, бесполезна. И "понятия" часто имеют несколько параметров, поэтому их нужно приделать к предикатам. Ещё, например, в теории множеств предикат быть множеством или быть классом (в какой теории вам нравится), выглядит (x = x). Уж очень хороший способ описать понятие множества!
Понятия изначально представлялись простейшим средством обсуждения семантики, и математика давно выросла из него. Есть теория моделей, которая описывает возможные семантики формальных теорий, используя богатые инструменты теории множеств.
Аргумент, почему с "есть" это не работает, я уже приводил.
> "селёдка - синоним для рыба"
Контрпример так себе, потому что природа непонимания происходит из того, что путаются "селедка" как имя класса и "селедка" как член класса. Первое позволяет использовать "есть" в смысле равенства, а второе в смысле принадлежности к классу. Это исправляется квантором "всякий", а не "с необходимостью".
"Селедка с необходимостью есть рыба" всё ещё может значить "селедка" синоним "рыбы" (но только с необходимостью!).
>> No.146940 Reply
File: _aa_src_77871_14271703515013.jpg
Jpg, 102.47 KB, 950×661 - Click the image to expand
edit Find source with google Find source with iqdb
_aa_src_77871_14271703515013.jpg
>>146850
> А как еще можно их рассматривать как эквивалентные? Постепенно?
Именно постепенно. В математике много равносильных утверждений, но нечасто увидишь, чтобы сразу в одном параграфе говорили о каком-то критерии.
>> No.146941 Reply
>>146935
А ещё в математической литературе не встречается семистопный ямб. Обращу внимание, что у нас семиотическое обсуждение.
Множество - это терм теории множеств. Понятие "множество" принадлежит поэтому не к самой теории множеств, а к метатеории. Множество - это набор символов, удовлетворяющий некоторым условиям. Чтобы описать эти условия (то есть описать, какие строки считаются термами), нужны предикаты. Метаязык ведь тоже может быть формальным. Если это твой единственный аргумент против моего определения понятия, то я не впечатлён.
О селёдке и рыбе я говорил именно как об именах классов, в твоей терминологии. Чтобы исключить нежелательное понимание, нужно явно оговорить, что такое "есть с необходимостью", "есть то же самое" и т.п. Причём сделать это нужно обязательно, потому что иначе слово "есть" будет источником ошибок.
>> No.146946 Reply
>>146941
> Множество - это терм теории множеств.
> Множество - это набор символов, удовлетворяющий некоторым условиям
У тебя в голове каша получилась, когда ты прочитал введение в мат. логику.
Терм в теории первого порядка для теории множеств - это объект, которые называет множество, а предикатные символы и формулы с одной свободной переменной называют предикаты (одноместные отношения). Множество есть денотат терма в теории множеств.
Действительно, чтобы установить соответствие между термами, предикатными символами, функциональными символами и объектами (в данном случае, множествами), отношениями (отношением принадлежности), функциями (этих не оказалось) нужно вложение в теорию множеств как метатеорию. Это соответствие называется отображением интерпретации, а последний набор объектов называется моделью формальной теории.
> Чтобы описать эти условия (то есть описать, какие строки считаются термами), нужны предикаты.
Ну есть у тебя рекурсивные определения формул и термов с вычислимым предикатом из метатеории, который это проверяет... Это само разумеющееся и никакое отношение к дискуссии не имеет. Видимо, потому что ты запутался в простых вещах вышеописанных.
> О селёдке и рыбе я говорил именно как об именах классов, в твоей терминологии
Врун.
> > "всякая селёдка с необходимостью есть рыба"
"Всякая селедка" указывает на неопределенного члена класса "селедка". Или в классе селедок содержатся не селедки, которые мы видим в магазине, а другие классы (!).
> Причём сделать это нужно обязательно, потому что иначе слово "есть" будет источником ошибок.
> > "Селедка с необходимостью есть рыба" всё ещё может значить: "селедка" синоним "рыбы" (но только с необходимостью!).
>> No.146947 Reply
>>146946
В теории множеств не предполагается наличие какой-то внеязыковой действительности. Вопросы интерпретации не имеют отражения в теории множеств - если теория формализована правильно и до конца. Множество - это строка символов, а не её денотат. Терм не называет множество, а является им. Аналогично и с функциональными символами. Причём я абсолютно не соглашусь с утверждением, что предикат является одноместным отношением. Отношение есть объект теории множеств, а предикат - объект из её метатеории.

Когда без слов "всякая" - название класса. И не обзывайся, ты тут не самый умный.
>> No.146952 Reply
File: Untitled.png
Png, 144.27 KB, 597×758 - Click the image to expand
edit Find source with google Find source with iqdb
Untitled.png
>>146947
Исправление чужих заблуждений для меня не является приоритетом. В своей правоте в данном случае я уверен. На этом разговор заканчивается.
Почитай §31 известной книжки Kleene ещё.
>> No.146954 Reply
File: p0038.png
Png, 211.83 KB, 702×1034 - Click the image to expand
edit Find source with google Find source with iqdb
p0038.png
>>146952
В свою очередь сошлюсь на книжку из Нанкаго.
Невежливо сначала влезать в разговор, а потом отказываться его продолжать.
>> No.146956 Reply
>>146954
> сошлюсь на книжку из Нанкаго.
Стало ещё глупее.
>> No.146960 Reply
File: macro-da-ty-ohuel.png
Png, 442.46 KB, 805×613 - Click the image to expand
edit Find source with google Find source with iqdb
macro-da-ty-ohuel.png
>> No.146961 Reply
>>146960
И не за чем Гусейна-Заде постить.
Всем известно, что первый том бурбаки - совершенно логически неосмысленный текст, полный ошибок, потому что из авторов никто ничего не понимал в логике.
Хотя так даже легче, потому что кто-то потратил время.
https://www.dpmms.cam.ac.uk/~ardm/logbanfinalmk.pdf
https://www.dpmms.cam.ac.uk/~ardm/inefff.pdf
>> No.146964 Reply
>>146961
Это известно только лапотникам. Если бы ты не слушал подлые наветы и всё-таки прочитал бы генерала, то понял бы, что у него всё написано идеально. Я уже, кажется, когда-то постил на Доброчане разбор нелепость обвинений бурбаков в непонимании матлогики, и даже конкретно этой pdf'ки про якобы определение единицы. В процессе обнаружил, что большинство критиков не понимают, что такое эпсилон-оператор Гильберта и почему он охуенен. Ackermann’s dual operator, my ass.
>> No.146970 Reply
>>146964
> что у него всё написано идеально.
Множество - терм. Лол. не слышал, чтобы для множеств были определены операции приписывания.
В любом случае, книжки абсолютно устарелые и бесполезные. Тратить на них время не собираюсь.
>> No.146983 Reply
File: macro-боже-с-кем-я-сижу-на-одной.jpg
Jpg, 57.47 KB, 510×340 - Click the image to expand
edit Find source with google Find source with iqdb
macro-боже-с-кем-я-сижу-на-одной.jpg
>>146970
Того не слышал, этого не встречал, Бурбаки не уважает.
>> No.146984 Reply
>>146983
Уж всяко лучше, чем не знать общепризнанную математическую логику.
Если ограничиться бесполезными книжками Бурбаки, то получится совершенные невежества (не знающие никаких логических результатов после 1928 года), вроде тех, что вопрошают тут, почему у остальных не было опыта сумасшествия. Лучше бы выучили простейшие понятия мат.логики.
>> No.146985 Reply
А могли бы теорию типов осваивать...
>> No.146988 Reply
>>146984
> не знающие никаких логических результатов после 1928 года
> Почитай §31 известной книжки Kleene
Лол.
> Лучше бы выучили простейшие понятия мат.логики.
Как будто кто-то их не выучил.
>> No.146995 Reply
>>146988
А пик был из манина, что написан не так давно.
> Как будто кто-то их не выучил.
Явно не выучили, потому что путают формулы/предикатные символы и сами предикаты.
>> No.146999 Reply
Нужны учебники по векторному и тензорному анализу, а также по обыкновенным дифференциальным уравнениям. Простые. Чтобы благодаря ним можно было решать практические задачи.
>> No.147006 Reply
Существуют ли такие численные методы решения дифференциальных уравнений, в которых имела бы место точная оценка ошибки сверху на данном шаге решения?
Правило Рунге не даёт достаточную точность.
>> No.147170 Reply
File: Безымянный.png
Png, 24.30 KB, 1000×290 - Click the image to expand
edit Find source with google Find source with iqdb
Безымянный.png
Привет, /u/.
Продолжаю заниматься приближенным нахождением 5-мерной функции по точкам.

Забил точки в вольфрам, но результат показался грубоватым (и жутковатым). Решил выделить вручную так много закономерностей, сколько смогу.

Выделил одну из переменных и получил такие вот графики для разных наборов фиксированных переменных.

Очень похоже на линейную зависимость (да и по смыслу, она здесь весьма к месту). Входные данные исследуемой функции точные, а вот результат (имею в виду значения f(x1,x2,x3,x4)) я могу получить только в округлённой до 3 знаков форме.

Характерны ли такие графики для искажений при округлении и можно каким-то образом по точкам восстановить исходную линию?
>> No.147180 Reply
File: tumblr_nwgta0ftO11u86t2qo1_540.gif
Gif, 1670.79 KB, 540×304 - Click the image to expand
edit Find source with google Find source with iqdb
tumblr_nwgta0ftO11u86t2qo1_540.gif
>>146999
> обыкновенным дифференциальным уравнениям
Филиппов сборник задач по дифференциальным уравнениям.
Все крайне просто, почти без глубокой теории, практические методы + примеры. Рекомендую юзать в комплекте с решебником, который легко гуглиться.
>> No.147220 Reply
доброаноны, помогите. Мне не понятно, что значит множество имеет Жорданову(И Лебегову) меру ноль. Объясните пожалуйста этот момент
>> No.147224 Reply
>>147220
Гм. Странный вопрос. А что такое "множество имеет жорданову меру три" - тебе понятно? Если это тебе тоже непонятно, то тогда ты просто не знаешь, что такое жорданова\лебегова мера. А вот если тебе понятно второе, но не первое, то я даже не знаю, как такое бывает.
Множество X имеет меру Жордана нуль, если для любого е>0 существует конечный набор отрезков I1, ..., In такой, что сумма длин этих отрезков меньше е, и объединение этих отрезков содержит Х.
Для определения лебеговой меры нуль нужно заменить "конечный набор отрезков" на "счётный набор отрезков".
В многомерном случае нужно заменить "отрезки" на "параллепипеды", а "сумму длин" на "сумму объёмов".
>> No.147243 Reply
сап, образовач.
учусь в техническом ВУЗе в ДС на 1-ом курсе.
проебываю, лень ездить, дорога длинная и долгая, поэтому, анон, посоветуй литературы по следующим предметам:
1) дискретка
2) линал
3) матан и аналитическая геометрия
заранее благодарю.
>> No.147252 Reply
>>147243
Ты думаешь, ты один такой с такими оригинальными запросами? Читай тред, сто раз уже всё рекомендовали-перерекомендовали.
>> No.147362 Reply
Как извлекать корни, к примеру в SL{2}(Z{3})? Тупое решение системы уравнений - это конечно не очень сложно, но хотелось бы знать, есть ли более быстрый способ.
>> No.147383 Reply
File: 16ac256d9f5b7a7b8788a4baa42cf487daef98d2.jpg
Jpg, 621.31 KB, 1000×1000 - Click the image to expand
edit Find source with google Find source with iqdb
16ac256d9f5b7a7b8788a4baa42cf487daef98d2.jpg
Можете пояснить или подкинуть методичку. Циркуляция поля, поток поля, интегралы второго рода, формулы Грина, Остроградского, Стокса. Не могу понять как делать...
>> No.147395 Reply
>>147362

enwiki://Square_root_of_a_2_by_2_matrix

В этой статье изложен некий явный метод для вещественных матриц. Понятно, что он будет работать и над конечным полем при условии, что соответствующие квадратные корни в поле существуют(иначе, в принципе, можно взять квадратичное расширение конечного поля). Думаю, ты понимаешь, что сходу на вопрос о том, даст ли этот метод все корни, тебе никто не ответит
>> No.147399 Reply
>>147383
Письменный. "Конспект лекций по высшей математике. Полный курс".
>> No.147406 Reply
File: tumblr_inline_nxqe15T9w81s1jww5_540.jpg
Jpg, 180.88 KB, 540×699 - Click the image to expand
edit Find source with google Find source with iqdb
tumblr_inline_nxqe15T9w81s1jww5_540.jpg
>>147399
Спасибо!
>> No.147445 Reply
File: 1387616574903.png
Png, 441.51 KB, 937×960 - Click the image to expand
edit Find source with google Find source with iqdb
1387616574903.png
Доброанон, поскажи пожалуйста сборники или одиночные интересные, не механические задачки по математике на логику, уровня 9-11 класса.
>> No.147446 Reply
File: intellectualfreedom.gif
Gif, 46.99 KB, 600×381 - Click the image to expand
edit Find source with google Find source with iqdb
intellectualfreedom.gif
Есть ли тут аспиранты или прошедшие по этому пути? Как проходит ваш рабочий день? Как проходят встречи с руководителем? Как выбирается тема исследовательской работы? Обязательно ли преподавать? Удастся ли совмещать аспирантуру с работой в смежной области? Как проходит защита кандидатской? Какие подводные камни, в общем?
>> No.147447 Reply
>>147446
И главный вопрос, зачем идти в аспирантуру, если можно спереть трактор на PhD?
>> No.147451 Reply
>>147447
У меня, например, диплом специалиста и плохой рейтинг, поэтому трактор - не вариант. При все этом есть предложение от дипломного руководителя пойти к нему аспирантом.
>> No.147453 Reply
>>147451
Окей, как насчет за два года получить магистра с хорошим рейтингом и попробовать трактор еще раз? Или если не совсем ущербный диплом бакалавра, то на магистра сразу за бугром? Опускаем момент с тем зачем специалисту с плохой успеваемостью вообще в науку лезть. Но поверь, няш, боль и унижение от всех подряд в Мордоре хуже чем боль и унижение только от от руководителя, а потом от грантодателей за границей.
>> No.147456 Reply
>>147453
> Окей, как насчет за два года получить магистра с хорошим рейтингом и попробовать трактор еще раз?
Зачем еще раз проходить одно и то же? Ведь предметы у магистров и спецов практически совпадают. Да и рейтинг по специалитету у меня безупречный, так как почти все предметы были с кафедры, куда я сознательно шел.
> Или если не совсем ущербный диплом бакалавра, то на магистра сразу за бугром?
Опять же, лишние годы, за это время можно уже и кандидата получить.
> Опускаем момент с тем зачем специалисту с плохой успеваемостью вообще в науку лезть.
Ты правда веришь, что успеваемость является надежной оценкой перспективности в научной работе? Может это и так, но только в идеальных универах, где преподы няшки, а все предметы интересные.
>> No.147475 Reply
>>147456
> диплом специалиста и плохой рейтинг
> рейтинг по специалитету у меня безупречный
Рожден быть ученым.
Во-первых, если ты упорно не хочешь признавать то что потерял год жизни и хочешь на основе этой ошибки строить свое светлое северокорейское будущее, то почему тебя волнуют какие-то проблемы, по сравнению с этой столь мизерные, что ими можно принебречь?
Во-вторых, успеваемость, суть которой - воспроизвести лекции, это хороший фактор отсеивающий совсем уж безнадежных.
В-третьих, няшки-преподы и возможность выбирать только интересные тебе предметы это просто нормальный универ какой-то нормальной страны.
>> No.147477 Reply
>>147453
> Есть ли тут аспиранты или прошедшие по этому пути?
Я, правда, по физике прошёл, но думаю, принципиальной разницы нет. Довольно свободный график, с редкими обязательными делами. На какие-то лекции/семинары (иняз, философия) надо было ходить на первом году аспирантуры, потом - почти свободен. Есть ещё педпрактика - вести какие-нибудь занятия, обычно в том же вузе, пол-года. Ерунда, по сути. С научруком видишься когда может он, по договорённости. Иногда пишешь/звонишь, особенно когда он в разъездах.

Экзамены ещё. Философия, иняз, и по специальности. Но это всё точечные дела.

>>147453
> боль и унижение от всех подряд в Мордоре
Анон, который спрашивал, ну про этого докладчика ты всё понел, я думаю.
>> No.147493 Reply
File: IMG_20151120_143642912.jpg
Jpg, 1039.41 KB, 1836×3264 - Click the image to expand
edit Find source with google Find source with iqdb
IMG_20151120_143642912.jpg
Анон, подскажи пожалуйста как выполнить 1 и 2 квест.

1 - найти каноническое и параметрическое уравнение уравнение прямой.

2 - найти точку пересечения прямой и плоскости.
>> No.147497 Reply
>>147493
Не подскажем. То, о чем ты спрашиваешь - самые первые страницы любого вузовского учебника геометрии. Например, в Александрове это ровно первая глава.
>> No.147523 Reply
File: arnold1.jpg
Jpg, 74.06 KB, 600×648
edit Find source with google Find source with iqdb
arnold1.jpg
File: ZrHgbsEKTHg.jpg
Jpg, 100.89 KB, 800×600
edit Find source with google Find source with iqdb
ZrHgbsEKTHg.jpg

>>147383
Ну твой идеал - Матматоды Арнольда, глава про дифференциальные формы. (Или в других местах про них). Потратишь чуть больше времени, чтобы врубиться, зато потом не будет проблем, не нужно будет запоминать кучу ненужной хрени.

Если вкратце, то есть отличный метод обращения со всем этим делом. символы dx, dy, dz (1-формы) можно складывать, умножать, дифференцировать и интегрировать. Если есть интеграл int P dx + Q dy по кривой, то просто заменяешь x = x(u), y = y(u) и применяешь цепное правило, чтобы получить обычный интеграл \int (P dx/du + Q dy/du) du , когда есть двойной и старшие интегралы правило таково:
dxdy = -dydx, в частности dxdx = -dxdx = 0 (все дифференциалы антикоммутируют, это отражает смену ориентации в интеграле). Поэтому,например, замена координат x, y -> u, v Будет выглядеть так:
dxdy = dx(u,v)dy(u,v) = (A du + B dv)(С du + В dv) = AB dudv + CD dvdu = (AB - CD)dudv, то есть обычная формула замены координа в интеграле через якобиан.

Если есть вектор v = (P, Q, R), то \int P dydz + Q dzdx + R dx*dy = \int (v, dS) поэтому заменой координат x, y, z = x(u,v), y(u,v), z(u,v) приводится к обычному интегралу в плоскости u, v.
Можно заметить, что при x-координате вектора P стоят только дифференциалы dy, dz и так далее. Тут важно следить за порядком умножения дифференциалов.
   Так же правило обобщается на большие размерности. (dxdydz = -dydxdz = dydzdx)

Осталось вывести формулы Грина, Гаусса-Остроградского и Стокса.
Для этого нужно уметь дифференцировать формы (это обобщает градиент, ротор и дивергенцию).
d(sumi vi dxi) = sumi sumj (dvi/dxk) dxk*dx_i
d(f(x,y)dx) = (df/dx)^dxdx + (df/dy)dydx = (df/dy)dy*dx. (Значок "d" на каждое слагаемое действует так: дифференцирует функцию, стоящую перед дифференциалами, вдоль каждого направления d/dxi, и умножает дифференциалы на dxi. В частности дифференциал функции - обычный дифференциал функции. df(x, y) = df/dx dx + df/dy dy). Стоит заметить, что в продифференцированной форме на один дифференциал больше.
А теперь есть очень важная общая формула Стокса (обобщает формулы Грина, Гаусса-Остроградского, Стокса и другие).
!!\int d w = \int w!!, где слева интеграл идёт по поверхности (n мерной поверхности), а справа по её границе (n-1 мерной границе).

Докажем для разминки формулу Грина:
w = P dx + Q dy
dw = (dP/dx dxdx + dP/dy dydx) + (dQ/dx dxdy + dP/dy dydy)=
Приводим слагаемые, вспоминаем, что dxdy = -dydx и dxdx = dydy = 0)
= (dQ/dx - dP/dy) dx*dy. Если теперь навесить знак интеграла, то получится формула Грина.


Ну и есть такое правило (можно подумать почему именно так), что вектору v = (P, Q, R) можно сопоставить 1-форму и 2-форму:
P dx + Q dy + R dz
P dydz + Q dzdx + R dx*dy.
Если продифференцировать их, применить общую формулу Стокса, то получатся теоремы
Стокса и Гаусса-Остроградского соответственно (про циркуляцию и поток векторного поля через поверхность).

В частности ротор вектора v - это вектор, соответствующий дифференциалу 1-формы:

d(P dx + Q dy + R dz) = (dP/dy dydx + dP/dz dzdx) + (dQ/dx dxdy + dQ/dz dzdy) + (dR/dx dxdz + dR/dy dydz) = (dR/dy - dQ/dz) dydz + (dP/dz - dR/dx) dzdx + (dQ/dx - dP/dy) dx*dy = rot (v).

А дивергенция вектора - это число, которое получается при дифференциировании 2-формы:
d(P dydz + Q dzdx + R dxdy) = dP/dx dxdydz + dQ/dy dydzdx + dR/dzdx*dy =
(чтобы привести к виду A dxdydz осталось заметить, что среди дифференциалов требуется 0 или 2 перестановки, значит знак не меняется: dydzdx = -dydxdz = dxdydz)
= (dP/dx + dQ/dy + dR/dz)dxdydz = div(v) dxdydz

Как поймёшь это порешай любые задачки на криволинейные интегралы из гугла - проблем быть не должно. Тут вся наука сводится к паре простых и красивых правил.
>> No.147549 Reply
Пожалуйста, скажите, как доказать эквивалентность утверждений.
>> No.147550 Reply
File: Снимок.PNG
Png, 18.24 KB, 763×113 - Click the image to expand
edit Find source with google Find source with iqdb
Снимок.PNG
>>147549
Отклеилось
>> No.147555 Reply
>>147497
А мне подскажете? Не ладится у меня с кривыми второго порядка - на парах я всё понимаю, но когда надо самому сделать - не могу. Задача такая: надо найти уравнение гиперболы, когда дана одна точка, принадлежащая ему, уравнение асимптоты и уравнение одной из осей. Каков вообще алгоритм действий?
>> No.147561 Reply
>>147555
Алгоритм - подумать! Самый банальный вариант - приводишь заменой координат к удобному виду, в котором ты знаешь как выглядит уравнение (асимптота в прямую под 45 градусов, а ось в горизонтальную, или как-нибудь ещё) и подставляешь точку. Или переводишь пересечение прямых в начало координат сдвигом (это сооствествует обнулению линейных членов), вспоминаешь, что кривой соответствует матрица (ax^2+2bxy+cy^2->((a, b),(b, c))), ось - её собственный вектор Av = c*v, а асимптота - ноль квадратичной формы v^T A v = 0, и находишь матрицу. А вспомнить это можно играясь с каноническими уравнениями, матрицами и зная, что у матриц не так уж много инвариантов.
>> No.147612 Reply
File: Sbornik_lektsiy_p...
Pdf, 0.53 KB, 595×842
Sbornik_lektsiy_po_obschey_teorii_otnositelnosti_S.pdf
File: 1413887400003.png
Png, 403.32 KB, 523×849
edit Find source with google Find source with iqdb
1413887400003.png

>>147523
Серьёзно?
Расшаривать дифгем в доброчанчике без TeX-а?
Мне кажется, невозможно.
>> No.147618 Reply
>>147612
> Тензором называется отображение, сопоставляющее произвольному базису набор чисел
Фу такую бякушку советовать.
>> No.147619 Reply
>>147618
Что не так-то?
>> No.147622 Reply
>>147619
Тензор - это вектор из некоторого векторного пространства. Зачем затемнять суть и рассказывать про какие-то наборы чисел?
>> No.147624 Reply
>>147622
А что такое вектор?
И, кстати, тензор не вектор. Если в пространстве — простите, многообразии нет метрики, то тензоры ранга (1,0) и (0,1) (ковекторы и векторы то есть) принципиально различны и не могут друг в друга переходить.
>> No.147626 Reply
>>147624
Разумеется, тензор - это вектор.
Вектор - это элемент векторного пространства.
Тензорное произведение векторных пространств - это векторное пространство с некоторыми свойствами.
Тензор ранга (m,n) на пространстве V - это вектор тензорного произведения m экземпляров пространства V на n экземпляров сопряжённого пространства к V.
Тензор ранга (1,0) - это вектор из V. Тензор ранга (0,1) - это вектор из двойственного к V.
>> No.147630 Reply
>>147626
> Тензор ранга (0,1) - это вектор из двойственного к V.
Индексы перепутал... Но не суть. Наверное, в различных пособиях по разному.
У векторов и ковекторов различные преобразования при переходе из карты в карту, а вектор, по определению, это отображение одной из карт в Rn с заданным преобразованием из карты в карту. А раз у ковектора преобразование другое, то вектором он являться не может.
>> No.147631 Reply
>>147630
Ковекторы образуют векторное пространство, не так ли?
>> No.147633 Reply
>>147631
Ага. Но это пространство не над заданным многообразием, поэтому нельзя определять ковекторы как векторы, если речь идёт об одном и том же многообразии. В противном случае само многообразие тоже можно назвать вектором, векторное пространство которого состоит из двух элементов — его самого и нуля, роль которого может играть пустое множество.
>> No.147634 Reply
>>147633
Так вот, я не утверждал, что я говорю об одном и том же пространстве. Я утверждал, что тензор не нужно называть набором чисел, это затемняет его суть. Тензор - это вектор из тензорного произведения.
Про многообразие комментировать не буду.
>> No.147636 Reply
>>147634
А, если уж желаете так, то опять же что тогда вектор? Он как раз отображение на набор чисел, разве нет?
Ковектор тоже.

Следовательно, тензор — векторное произведение отображений на набор чисел, и в свою очередь тоже является таким отображением.
>> No.147637 Reply
>>147636
Нет, вектор - не набор чисел. Это элемент модуля над полем.
>> No.147639 Reply
Набор чисел появляется только тогда, когда зафиксирован какой-либо базис. До этого, вектор — не более, чем элемент векторного пространства.
>> No.147667 Reply
Привет, доброанон, у меня пробелема: мне 20 и я не знаю как оперировать дробьями. Не мог бы ты разжевать эту простоту или посоветовать книгу для очень слоупочных детей?
>> No.147669 Reply
>>147667
Любой учебник математики за шестой класс же. Или за пятый.
http://www.alleng.ru/edu/math1.htm
>> No.147765 Reply
File: a.png
Png, 5.02 KB, 534×43 - Click the image to expand
edit Find source with google Find source with iqdb
a.png
Аноны, кто-нибудь разбирается в анализе? Я не могу понять, что мне делать с таким выраженим. Оно вообще имеет смысл?
Нужно исходя из него сделать какой-то вывод о функции f
>> No.147773 Reply
>>147765
Это определение предела функции действительной переменной в точке. Я тоже не знаю, что тебе с ним делать. Можешь пофапать на него, например.
>> No.147774 Reply
>>147773
Нет, это не определение предела.
>> No.147778 Reply
>>147774
И правда, я был невнимателен. Тогда действительно какая-то ебанутая запись. Если там нет опечаток, то: Для точки x0 существует дельта-окрестность (x0-\delta, x0+\delta), за пределами которой значение функции не совпадает со значением в x0. Но если они правда хотели сказать именно это, то они использовали самую идиотскую запись, которую только можно придумать. Скорее всего там опечатка.
>> No.147780 Reply
>>147778
Там нет опечатки, формулировка именно такая. %% капча фейл одном ветра %%
>> No.147818 Reply
>>147765
>>147765
Какой вывод нужно сделать про функцию? Выражение осмысленно, смысл у него такой: существует открытый интервал, содержащий точку х0, за пределами которого все значения функции отличаются от х0 довольно сильно.
>> No.147820 Reply
>>147765
Перевожу с рунического:
Существует такое число эпсилон больше нуля и такое число дельта больше нуля, что:
ЕСЛИ взятая по модулю разница между двумя числами х и х0 больше за дельта нежданчик, о х и х0 нигде раньше не упоминалось ТОГДА получается, что взятая по модулю разница между значением функции в точке х и значением функции в точке х0 больше эпсилон.

1. Это не определение границы по Коши, там разница меньше эпсилон.
2. Это не определение возрастающей функции

Так что анончик, смотри контекст
>> No.147854 Reply
Мы писали курсачи всем чаном...
>> No.147864 Reply
Добрый день, анон.
мне 21 и я нихрена не понимаю в математике.
как можно подтянуть знания? моя будущая профессия связана с химией, а в аналитике и электрохимии довольно много матана
>> No.147866 Reply
>>147864
Математика бывает сильно разная. Разница между чистой и прикладной больше, чем между физикой и химией. Тебя что конкретно интересует?
>> No.147871 Reply
>>147866
Полагаю, что матан, а затем теорвер/матстат, раз химик.
>> No.147887 Reply
У предельного перехода есть какое-нибудь словесное определение?
>> No.147888 Reply
>>147887
Предел функции в точке - точка, которая делает функцию непрерывной.
>> No.147985 Reply
Привет, посоветуйте хороших учебников по аналитической геометрии для первокурсоты.
>> No.147986 Reply
>>147985
Что значит "хороших"? Уточни, чего ты ждёшь. Мне хорошо одно, тебе другое.
>> No.147988 Reply
>>147866
любая. суть в том что я просрал все математические полимеры еще в 9 классе и успел забыть как извлекать дискриминант.
по сути дела нужна прикладная математика, с синусами и косинусами для построения/чтения графиков. ну или понятие о том какая функция зачем нужна, чтобы я смог нормально по полной использовать свой йоба-калькулатор за 2300 (калькулятор 10/10, купил бы снова. двухстрочный со скобками и все такое)
>> No.147990 Reply
>>147988
Демидович, Краткий курс высшей математик.
>> No.147995 Reply
z^8=1
?
>> No.148000 Reply
>>147995
cos pi/4 = 2 * cos^2 pi/8 - 1
>> No.148002 Reply
>>147986
Ну например кривые второго порядка, чтоб разжевано было, как например найти фокусы эллипса, если он сдвинут и перевернут. Или задачник с примерами решений было бы гораздо лучше и нагляднее.
Заранее извиняюсь за мой русский.
>> No.148047 Reply
Зачем в матанализе понятие базы?
>> No.148048 Reply
>>148047
Смотря какой базы. Их вообще-то дофига.
>> No.148065 Reply
File: Horo018.jpg
Jpg, 245.03 KB, 850×1147 - Click the image to expand
edit Find source with google Find source with iqdb
Horo018.jpg
Допустим есть один интеграл:
((sin(x))^n)*(cos(x))^m
x=0..2pi
n, m - натуральные.
Этот определенный интеграл всегда равен нулю, не зависимо от н и м? Я думаю да, но не очень уверен.
>> No.148067 Reply
>>148065
Nyet. Если n и m четные, то оба множителя всегда положительные. Не будет у тебя интеграл от функции, расположенной выше нуля равняться нулю.
>> No.148068 Reply
>>148065
> Этот определенный интеграл всегда равен нулю, не зависимо от н и м?
Нет, конечно. Например, при четных м и н подынтегральная функция всюду положительна, кроме концевых точек. Кажется, явная формула для этого интеграла была в задачнике Демидовича. Если хочется вывести самому, то можно проинтегрировать по частям и получить, что интеграл ((sin(x))^n)(cos(x))^m совпадает с интегралом ((m-1)/(n+1))((sin(x))^(n+2))*(cos(x))^(m-2). Уменьшая m на двойку, придем к m=1, либо m=0. Первый случай тривиален(подынтегральная функция становится производной), второй разрешается при помощи ещё одного применения интегрирования по частям((sin x)^k dx = - (sin x)^(k-1) d(cosx)).
>> No.148077 Reply
File: верно подмечено.jpg
Jpg, 86.55 KB, 1006×566 - Click the image to expand
edit Find source with google Find source with iqdb
верно подмечено.jpg
>>148068
>>148067
Мм, ну мне интересны только те случаи когда интеграл ноль. Если оба нечетны, то точно ноль?
>> No.148082 Reply
>>148077
> Если оба нечетны, то точно ноль?
А ты посчитай сам. Ход решения тебе, тащемта, объяснили.
>> No.148083 Reply
>>148082
Ну ок. Спасибо!
>> No.148182 Reply
File: kjno.PNG
Png, 14.66 KB, 643×66
edit Find source with google Find source with iqdb
kjno.PNG
File: O73hQR7R-o.jpg
Jpg, 137.37 KB, 1280×720
edit Find source with google Find source with iqdb
O73hQR7R-o.jpg

Юнач, помоги с заданием. Точнее, помоги довести до ума. Пикрелейтед само задание и моя попытка решения. Я нашел все вершины эллипса, фокусное расстояние и сами фокусы (а т.к. эллипс и гипербола имеют общие фокальные хорды, значит, всякие фокусы, фокусные параметры и прочая фигня у них общая). Теперь требуется составить уравнение этой гиперболы. Все, на этом я потратился.
>> No.148191 Reply
Кто может пояснить алгоритм Риша?Я гуглил но нашел только материалы на англиском.
>> No.148195 Reply
Расскажите про лемму змея.
>> No.148200 Reply
>>148191

Da ti ohuel. Алгоритм Риша в подробном изложении занимает 100 страниц и требует для своего понимания хорошего знания дифференциальной алгебры(которого у тебя наверняка нет). Ты предполагаешь, что его кто-нибудь возьмет и разжует для тебя? Подозреваю, что незнание английского - наименьшее из препятствий, стоящих между тобой и пониманием алгоритма Риша.

>>148195

Формулировка и полное доказательство лежат в вики.
https://en.m.wikipedia.org/wiki/Snake_lemma
>> No.148201 Reply
>>148182
Так все из картинки уже следует. Гипербола пересекает эллипс в тех же точках, что и фокальные хорды. Найди точку пересечения эллипса(достаточно одной, остальные отличаются только знаками) с фокальными хордами и подставь её координаты в уравнение гиперболы. Это даст одно уравнение для коэффициентов гиперболы. Второе получишь из уравнения фокального расстояния гиперболы.
>> No.148229 Reply
File: Смесь.jpeg
Jpeg, 78.21 KB, 1362×651
edit Find source with google Find source with iqdb
Смесь.jpeg
File: Арксинус.jpeg
Jpeg, 32.41 KB, 1362×651
edit Find source with google Find source with iqdb
Арксинус.jpeg

http://pastebin.com/djYyN5Ev

Добрый вечер.
Я пишу код на R на первых n страницах li/s/p такой темы обнаружено не было, поэтому отпишусь сюда. Статистика же, но языка не знаю почти полностью и не понимаю, что происходит.
В программе происходит генерация выборки из смеси распределений, потом подсчет всяких параметров и рисование обычной и эмпирической функций распределения. В основном для того, чтобы узнать, нет ли ошибки. А ошибка есть: ЭФР и ФР совсем не похожи, дисперсия чуть не вдвое отличается и не попадает в доверительные интервалы.
Забавность в том, что почти аналогичная (написанная о мотивам первой за 10 минут) программа на Maple работает превосходно и на выборке из 1000 элементов дает погрешность меньше 0.02.

Пытался найти ошибку. Распределение арксинуса (rasin) написано хорошо, смотрите фото. Получается, что ошибка в функции { return ( 0.6rnorm(N, mean = 0, sd = 3) + 0.4rasin(N) )}. Но там одна строчка, какая может быть ошибка?
В общем, котаны, посмотрите код и ткните носом, пожалуйста.
Также не против получить замечания типа "Нахрена ты моменты (доверительные интервалы, параметры асимметрии и прочее) писал, это делается одной функцией так-то и так-то". Просто мне было легче написать 2 строчки, чем искать подходящие функции в интернетах.
>> No.148237 Reply
File: rstudio.png
Png, 9.79 KB, 250×88 - Click the image to expand
edit Find source with google Find source with iqdb
rstudio.png
>>148229
Не разобрался, что не так с эмпирической функцией распределения, но дисперсию ты посчитал неправильно - D(ax)=a^2D(x) и 0.6^29 + 0.4^24.5 = 3.96
По поводу моментов - есть var(x) для variance, старших моментов вроде нет в базовых пакетах. Такие интервалы тоже ручками считать, если будешь юзать какие-то стат. модели типа lm(), то там они обычно в summary() функции расчитываются.
Ну и просто замечания - я бы вместо самопального метода половинного деления заюзал встроенную функцию для численной оптимизации optim. Вместо цикла for юзал *apply и делал функции векторизованными сразу. И как-то принято оператор присваивания писать не (=), а (<-), помнится, есть вероятность какие-то грабли словить на этом.
>> No.148238 Reply
>>148237
Но в данном случае коэффициенты стоят в функциях распределения, а не перед случайной величиной хотя то, что 3.96 замечательно укладывается в полученные мной данные, намекает на верность ответа.. Должно считаться так (с учетом нулевого матожидания):
DX = int(x^2*dF(x)),
F(x) = 0.4(функцию распределения арксинуса) + 0.6(функцию распределения нормальной случайной величины).
Разве нет?
>> No.148239 Reply
File: капча.png
Png, 3.25 KB, 253×48 - Click the image to expand
edit Find source with google Find source with iqdb
капча.png
>>144684
Анон, как тебе задания десятой студенческой олимпиады по алгебре на мехмате?
http://halgebra.math.msu.su/Olympiad/2015/variant.pdf
Можешь ли отсортировать задачи по сложности?
Верно ли, что вторая задача слишком простая?
>> No.148240 Reply
File: rstudio.png
Png, 9.79 KB, 250×88 - Click the image to expand
edit Find source with google Find source with iqdb
rstudio.png
>>148238
Изначально ты производишь действия со случайными величинами, их умножаешь и складываешь. Вангую, проблема в том, что эти преобразования отражаются на плотности и функции распределения не так прямолинейно, смотри хотя бы на вики, в самом низу статьи ruwiki://Плотность_вероятности
>> No.148241 Reply
>>148240
Ура, я осознал свою ошибку и понял, почему в мэйпле все было совсем иначе.
По сути, я беру нифига не смесь (т.е. случайную величину, имеющую функцию распределения 0.4F1+0.6F2), а сумму двух случайных выличин 0.4X+0.6Y, где X имеет распределение арксинуса, а Y — нормального распределения.
В первом случае в дисперсии коэффициенты вычисляются умножаются без квадратов, а во втором с квадратами.
Сейчас исправлю и, если график будет посимпатичнее, запощу.
>> No.148242 Reply
File: Исправлено.jpeg
Jpeg, 64.73 KB, 1362×651 - Click the image to expand
edit Find source with google Find source with iqdb
Исправлено.jpeg
>>148241
Как же отвратительно выглядят мои выходные данные. Надо бы обучиться, как красиво выводить, без всяких [1] и ""

[1] "F - смесь (выпуклая комбинация) двух распределений, "
[1] "параметр смешивания p1 = 0.4."
[1] "Смешиваемые распределения:"
[1] "F1 -- Арксинус-распределение с параметром d = 3;"
[1] "F2 – нормальное (0, 9)."
[1] ""
[1] "Мат.ожидание смеси: 0"
[1] "Выборочное среднее выборки:"
[1] -0.01112802
[1] "Дисперсия: 7.2"
[1] "Выборочная дисперсия:"
[1] 7.849583
[1] "доверительня вероятность 0.9"
[1] "МО" "-0.156858547515579" "0.134602506942459"
[1] "Дисперсия" "7.28069726411445" "8.41846774785421" А вот и нет! Был шанс 0.1 ошибиться, это и произошло
[1] "доверительня вероятность 0.95"
[1] "МО" "-0.184776653901627" "0.162520613328507"
[1] "Дисперсия" "7.17171393024496" "8.5274510817237"
[1] "доверительня вероятность 0.9974"
[1] "МО" "-0.277936407830048" "0.255680367256928"
[1] "Дисперсия" "6.80804812765837" "8.89111688431029"
>> No.148247 Reply
File: rstudio.png
Png, 9.79 KB, 250×88 - Click the image to expand
edit Find source with google Find source with iqdb
rstudio.png
>>148242
> Как же отвратительно выглядят мои выходные данные. Надо бы обучиться, как красиво выводить, без всяких [1] и ""
?print выдаст страницу справки, там функция unquote упоминается. Вообще, если хочешь красиво все оформить, рекомендую освоить Rmd - местный literate programming Это когда код и текст вместе в файле, причем первый прямо влияет на второй. Полагаю, ты в RStudio ебашешь, тогда там прям выбираешь создать Rmd документ и он тебе шаблон сгенерит, посмотришь как он красиво все отрендерит. Там же справка есть. Плюс рекомендую для графиков юзать ggplot2, он и красивее, и после некоторого привыкания удобнее, особенно если строить сложные графики из нескольких слоев.
>> No.148294 Reply
File: Jj3BVZC0Vnk.jpg
Jpg, 280.09 KB, 1224×1632
edit Find source with google Find source with iqdb
Jj3BVZC0Vnk.jpg
File: yui_hirasawa_png_...
Png, 69.85 KB, 328×363
edit Find source with google Find source with iqdb
yui_hirasawa_png_1_by_dawnkresher11-d4y5o5d.png

Доброчан, а как решать такие пределы? Использовать правило Лопиталя не могу, к сожалению. У меня есть типовые варианты, которые мне могут встретиться на контрольной, но хотелось бы понять как решать все пять примеров, чтобы решить и все остальные варианты.
>> No.148305 Reply
>>148294
Используй разложение в ряд Тейлора.
>> No.148309 Reply
Мне к наступлению ЕГЭ надо научиться безотказно решать тамошние задачи по стереометрии и планиметрии, чего я совершенно не умею. Какие материалы и разборы решения задач для этого посоветуете? Если просто решать из учебника, то упрусь в такую, которую решать не умею и не смогу ничего дальше делать.

В ЕГЭ-треде все вымерли, поэтому спрашиваю здесь.
>> No.148313 Reply
File: Screenshot_2015-12-16-18-14-21_com.vkontakte.andro.png
Png, 888.12 KB, 1080×1920 - Click the image to expand
edit Find source with google Find source with iqdb
Screenshot_2015-12-16-18-14-21_com.vkontakte.andro.png
Что требуется в задачи 1, я не понял, и какого вида граф в 3 задаче, что за sv ? Я к другому обозначению привык, заранее спасибо
А то промахнулся
>> No.148314 Reply
File: 1338136804543.jpg
Jpg, 152.56 KB, 1400×799 - Click the image to expand
edit Find source with google Find source with iqdb
1338136804543.jpg
Здравствуйте, матаноны. Недавно наткнулся на рассказ, который читал еще в детском возрасте, "Сатанинскую бутылку" Стивенсона. Теперь-то я знаю, что существует одноименный парадокс, но, хотите - верьте, хотите - нет, я про него подумал независимо во время прочтения. Хотя это вообще не принципиально. Принципиально то, что я, видно, тупой, и не понимаю, как он разрешается. Поэтому прошу здесь помощи, несмотря на то, что в этом треде я не пришей кобыле хвост с подобными просьбами.
Чтобы людям не приходилось гуглить, перескажу суть парадокса.
Существует бутылка, в которой живет черт. Бутылка может исполнить любое адекватное желание, помимо продления жизни. Однако человек, умирающий с этой бутылкой на руках, отправляется в ад. Как же от нее избавиться? А избавиться от нее можно лишь продав, причем обязательно с убытком для себя. То есть если вы купили эту бутылку у предыдущего владельца за 100 баксов, то продать вы ее можете лишь меньше, чем за 100 баксов. Но это все присказка.
Предположим, что вам предлагают бутылку за 1 цент. Ее, безусловно, не стоит покупать, вы банально не сумеете ее продать (надо пояснить, что в данном случае рассматривается идеализированная ситуация, то есть 1 цент - это универсальная минимальная единица материального блага). Теперь же вам предлагают бутылку за 2 цента. Все еще не стоит идти на подобную сделку, ведь вам-то ее придется продавать за 1 цент, а мы только что выяснили, что за 1 цент ее никто не купит. 3 цента? Может быть... Нет, не стоит. Ведь продать вы ее сможете за максимум 2 цента, а мы только что выяснили, что за 2 цента ее никто не купит. Об одноцентовиках и говорить тогда нечего.
Рассуждая так дальше, получается, что бутылку никто никогда не купит. Где ошибка в данном рассуждении?
P.S. Этот "парадокс" роднят с "парадоксом внезапной казни". Однако там я вижу ошибку в рассуждениях, а здесь нет. Ментальный блок, видно.
>> No.148315 Reply
>>148314
А почему ты думаешь, что здесь есть парадокс? Где он? Какие две вещи друг другу противоречат?
>> No.148317 Reply
>>148315
Здравый смысл подсказывает, что даже в идеальной ситуации за условный доллар бутылку можно купить риска для себя. Я понимаю, что в математике здравый смысл не аргумент, я и сам думал о том, что, возможно, парадокса и нет, но интернет мне сказал, что это именно что парадокс, схожий по механике с парадоксом внезапной казни.
>> No.148320 Reply
>>148317
Чтобы понять, покупать бутылку или же не покупать, нужно проделать рассуждение, которое ты только что проделал. Но ты не учитываешь человеческую глупость. ruwiki://Теория_большего_дурака
В целом, тут скорее не парадокс, а когнитивное искажение. Некоторые люди не думают перед тем, как совершают свои действия.
>> No.148335 Reply
>>148314
>>148317
>>148320
О, занятная тема. У меня тоже есть некоторое ощущение, что я совсем не понимаю какого-то важного глубинного лол смысла, который затрагивается этими парадоксами. Моя логика:
Парадокс неожиданной казни. На мой взгляд результат можно сформулировать как: если заключённый поверил в невозможность казни - она стала неожиданной, а следовательно начальник тюрьмы - честнейший и весьма тонкий весельчак. Если же не поверил - казнь оказалась ожидаемой, а начальник - лжец, негодник и толстяк. Проблема в том, что логически невозможно определить поверит ли заключённый, а следовательно и логического решения у задачи нет, как и самого парадокса. То есть в реальности его уверенность зависит от туевой хучи факторов, а всякие размышления на тему "абсолютно логичного существа" - это уже есть модели реальности разной степени достоверности.
Сатанинская бутылка. Тут всё завязано на планке "это достаточная цена, чтобы я смог впарить бутылку лоху". Планка абсолютно разная для каждого человека, и рано или поздно следуя рассуждениям "за 2 цента её никто не купит, а что если за 3?" - человек придёт к комфортной для себя сумме. Я не могу понять, почему они считают эту последовательность бесконечной, а саму задачу парадоксом.
Если мы говорим о людях, конечно. Есть смутное ощущение, что мы говорим о каких-то сферических существах в вакууме, но тогда для решения задачи эти существа должны быть тоже охарактеризованы, потому что всё упирается в итоге в 1 неизвестный фактор - поверил ли заключённый, и какая комфортная стоимость бутылки. По мне так это сорт оф софистика.
>> No.148337 Reply
>>148314
> Рассуждая так дальше, получается, что бутылку никто никогда не купит.
Купить за два цента, пожелать понять весь маркетинг и впарить лоху за цент. Профит!
>> No.148339 Reply
>>148305
Да и без него можно обойтись, вобщем-то...
>> No.148346 Reply
>>148339
Объясни как? Ряды Тейлора мне не подойдут.
>> No.148351 Reply
>>148335
Выпускаем свои собственные денежные единицы. Например, клочок бумажки с крестиком. Каждый раз когда хотим продать, рисуем новую ассигнацию, это чуть снижает стоимость сей валюты, итого продаём бутылку за одну бумажку по кругу, инфляция - наш друг.

мимо
>> No.148352 Reply
File: sup.pdf
Pdf, 0.07 KB, 595×842 - Click the image to get file
sup.pdf
>>148346
> Объясни как?
Держи первый и третий. Принцип понятен?
>> No.148361 Reply
>>148314
Плюс ещё не учитывается люди, которым всё равно на попадание в ад, или люди, которые сами не знают механизма работы бутылки. Всё как с рыночком короче, в теории всё идеально и никаких кризисов в принципе происходить не может, потому что слишком уж идеализированная, абстрагированная ото всего модель.
>> No.148402 Reply
>>148351
Годнота. Благодаря ВВП бутылкой можно попользоваться лет 10 и продать за ту же стоимость. Инфляция же.
Дьявол тупой гуманитарий, бутылка говно для лохов.
>> No.148407 Reply
>> No.148421 Reply
>>148352
Чорт, там в первом пределе не x стремится к бесконечности, а n. Ну ты понел.
Я страшно тупил, пока набивал эти решения, ибо уставший был, так что пардон.
>> No.148431 Reply
File: Снимок.PNG
Png, 12.13 KB, 458×76
edit Find source with google Find source with iqdb
Снимок.PNG
File: 2018de0145aafc8e1...
Jpg, 785.03 KB, 777×1087
edit Find source with google Find source with iqdb
2018de0145aafc8e1a0c40c31b37b280.jpg

Котаны, объясните как решить задание. А то так и не осилил векторы. С меня лоли.
>> No.148454 Reply
>>148431
Ищи вектор в виде x=k*[a1 x a2]. Он уже перпендикулярен всему, чему надо, а k находится из условия.
>> No.148485 Reply
File: 63v1.png
Png, 3.83 KB, 82×20 - Click the image to expand
edit Find source with google Find source with iqdb
63v1.png
Доброанон, что значит такая запись функции? Отсюда http://www.wireless-e.ru/articles/technologies/2006_1_63.php
>> No.148489 Reply
>>148485
Это не запись функции, это запись размерностей кода. n-мерный код, кодирующий k символов, как легко видеть из контекста в твоей ссылке:
> > Каждая строка матрицы представляет собой кодовое слово C=(n,k) и состоит из k информационных символов и n-k проверочных.
>> No.148495 Reply
>>148431
>>148454
Лучше разобраться всё-таки по-хорошему что значит перпендикулярен. Тогда всё станет очевидно.
>> No.148500 Reply
Помогите анончики как найти символ Лежандра (7/p)
>> No.148501 Reply
Помогите анончики как найти символ Лежандра (7/p)
>> No.148515 Reply
File: CodeCogsEqn.gif
Gif, 1.08 KB, 306×18 - Click the image to expand
edit Find source with google Find source with iqdb
CodeCogsEqn.gif
Помогите доказать секвенцию, пожалуйста.
>> No.148517 Reply
File: fast_aussie_van_1_1.jpg
Jpg, 91.27 KB, 830×780 - Click the image to expand
edit Find source with google Find source with iqdb
fast_aussie_van_1_1.jpg
>>144684
Насоветуйте пожалуйста хорошие учебники\пособия по математике, чтобы подготовиться к экзамену. Дело в том, что как она закончилась на первом курсе шараги, получил свой зачёт и забыл про неё аж на 4 курса. Сейчас внезапно понадобилась, потому что готовлюсь поступать по внутренним экзаменам, увидел это(пикрелейтед), решил только первое неравенство и приуныл.
не кидайте тапками, что-то мне подсказывает что задания не такие уж и сложные, просто я разучился их решать.
>> No.148525 Reply
>>148517
Мне эти задания до жути напомнили задачи из какого-нибудь сборника по подготовке к ЕГЭ (С часть). Предлагаю отталкиваться от этого.Надрачивал эту херню для ЕГЭ лет 5 назад
>> No.148532 Reply
>>148525
А есть какой-нибудь конкретный годный сборник? Или просто найти первый попавшийся и читать\решать его?

Просто вообще не знаю от чего отталкиваться, а времени всё меньше и меньше.
>> No.148827 Reply
>>148532
Ткачук, вроде, с избытком покрывает всю школьную математику.

Аноны, а может кто-нибудь у меня задачки попроверять? В основном очень базовый вычислительный линал. Позже ещё может коммутача какого нарешаю. Пока вот есть:

1) Задайте линейными уравнениями векторное пространство в арифметическом пространстве R^3, порожденное векторами
а) (1,1,1) и (1,1,-1); б) (1,1,1) и (2,3,4)

2) Предложите набор векторов, порождающих в R^3 векторное подпространство
а) x + y + z = 0; б) x + 2y + 3z = 0

1.a) x - y = 0; 1.б) x - 2y + z = 0; 2.а) (2,-1,-1) и (1,7,-8); 2.б) (3,0,-1) и (1,1,-1)
>> No.148851 Reply
File: slaboumie.jpg
Jpg, 36.68 KB, 700×444 - Click the image to expand
edit Find source with google Find source with iqdb
slaboumie.jpg
>>148827
Аноны-аноны, а расскажите, пожалуйста, где можно доступно почитать/посмотреть/послушать про тензоры и все с ними связанное. А то кругом физика, в ней они, а я ННП. И когда пытаюсь что-то посмотреть в интернетах по теме, то тоже совсем не вкуриваю. Откуда начать?
>> No.148852 Reply
>>148851
https://en.m.wikipedia.org/wiki/Tensor_product - более простого объяснения я не знаю.
Hint: когда в физике говорят о тензорах, то в 90% случаев имеется в виду просто-напросто матрица.
>> No.148853 Reply
>>148852
С теми, которые матрицы, все понятно (хотя эта их нотация с верхними и нижними индексами пока что смущает), а вот как работают остальные — ¯\(o_°)/¯. Кстати, аналогичный реквест про ротор-дивергенцию-градиент-етц. В случае декартовой системы координат я с ними работать умею, а вот откуда оно взялось и как работает в принципе — тотальный непонимат. За статью спасибо, почитаю.
>> No.148873 Reply
>>148853
> В случае декартовой системы координат я с ними работать умею
Так и всё, в принципе. Больше ничего на уровне определений про эти впщи знать не нужно. Можно, конечно, научиться писать их в сферических, циллиндрических и эллиптических координатах, но зачем? Работать по-настоящему они начинают позже, когда через них записываются разные варианты формулы Стокса(формула Гаусса-Остроградского и проч.).
>> No.148943 Reply
У меня не получается из нижеизложенного интеграла вывести бета-функцию заменой sin^2 x = t
S[п/2][0](sin^(p-1)x cos^(q-1)x)dx
Должно получиться 0.5B(p/2,q/2)
Я делаю так:
S[п/2][0](sin^(p-1)x cos^(q-1)x)dx
S[п/2][0]((sin^(p)x cos^(q)x)/(sin x cos x))dx
замена sin^2 x = t
Получаю
S[1][0]2(t^0.5p (1 - t^0.5q))dt
Синус и косинус снизу дроби убираются посредством появления 2sin x cos x от дтфференцирования sin^2 x
ЧЯДНТ?
>> No.148953 Reply
>>148943
.бамп
>> No.149028 Reply
Поверхностный интеграл второго рода и векторное поле это одна кухня?
>> No.149032 Reply
>>149028
Переформулируй, я не понимаю твоего вопроса в достаточной степени, чтобы развернуто на него ответить.
Связаны ли эти понятия? Ну связаны. Ты это хотел услышать?
>> No.149173 Reply
Котаны, как из расслоения Хоппфа следует, что pi_2(S^2) = Z?
Если написать кусочек гомотопической последовательности, имеем:
pi2(S^3) -> pi2(S^2) -> pi1(S^1) = Z -> pi1(S^3) = 0.
Последовательность точна.
Ядро pi1(S^1) -> pi1(S^3) будет все Z.
Значит, образ pi2(S^2) -> pi1(S^1) тоже все Z. Значит, сюръективный гомоморфизм. А инъекцию где взять?
>> No.149185 Reply
>>149173
Так ведь pi2(S^3) = 0
>> No.149210 Reply
>>149185
Нет, pi2(S^3) вычисляется позже и не представляется известной на данный момент.
Пока известными предполагаются лишь pi1. Остальное как раз вычисляется с помощью накрытий.
С цепочками 0 -> A -> B -> 0 и в самом деле все ясно.

Также почитал дополнительную литературу (как же мне не понравился Фоменко-Фукс. Вот Хатчер годнота, но там нужного я не нашел), появились еще вопросы.
Как строится S^inf и CP^inf? Я знаю лишь как через R^inf, то есть считать S^inf множеством всех сходящихся к 1 последовательностей. Но тут геометрия а не фан.
Что-то типа считать S^i вложенной как экватор в S^{i+1}? Тут возникает вопрос о стягиваемости. Интуитивно стягиваемость вроде понятна: вкладывая меньшую сферу в большую, мы делаем ее стягиваемой через большую сферу. Итого для каждой фиксированной сферы размерности n она стягивается начиная с S^{n+1}. Бесконечномерную определим как объединение всех вложенных друг в друга сфер. Это так или я несу чушь?

Вообще, пока (после одного дня поиска, ага) из книг наиболее годным оказался Хатчер. Но и он сложноват, хотелось бы найти что-нибудь типа Прасаловской "Комбинаторной геометрии", то есть книгу для умственно отсталых детей с миллиардом примеров и решением этих примеров . Есть такие?
>> No.149227 Reply
>>149210
Ээээ, pi2(S^3) = 0 очевидно из клеточного разбиения (S^3 = R^3 + point) Im(S^2) меньше S^3, значит найдётся точка вне образа. То есть отображение пропускается через отображение в плоскость, которое гомотопически тривиально

мимокрок
>> No.149228 Reply
>>148853
Там где-то повыше есть про это довольно подробный пост
>> No.149231 Reply
>>148852
> имеется в виду просто-напросто матрица
ШТОБЛЯТЬ?

мимо недовольный физик
>> No.149233 Reply
>>149231
Ок, просто-напросто матрица, инвариантная относительно замены координат.
>> No.149234 Reply
>>149231
В физике тензор определяют как "набор чисел". Это смешно.
>> No.149235 Reply
>>149227
Отлично, почти все понятно. Только вот это
> Im(S^2) меньше S^3
Вызывает сомнения. Мне кажется, можно придумать отображение, образ его замыкание которого покроет S^3, как прямая покрывает квадратик. Но тут нужно что-то типа теоремы о том, что такая "плохая" петля гомотопна "хорошей" с небольшим образом.
Но суть я уловил, благодарю.

Вопрос о стягиваемости бесконечномерной сферы остается.
>> No.149236 Reply
>>149233
Инвариантна физическая характеристика, которую описывает тензор, сам же тензор изменяется при смене базиса.
>>149234
Как "набор чисел" в физике даже вектор не определяется что логично, ведь это частный случай тензора
>> No.149237 Reply
File: тензор.png
Png, 73.25 KB, 701×585 - Click the image to expand
edit Find source with google Find source with iqdb
тензор.png
>>149236
Ну-ну.
>> No.149238 Reply
File: 340.jpg
Jpg, 375.96 KB, 2111×901 - Click the image to expand
edit Find source with google Find source with iqdb
340.jpg
>> No.149239 Reply
>>149235
Не придумаешь :P Непрерывный образ компакта - компакт (внутри метризуемой S^3 равносильно замкнутости). А во всю сферу не получится, например, поскольку разная размерность (Хаусдорфова топологическая, та, что через разбиение на шары). Кстати, это можно как-то проще понять?

Даже если бы смог придумать с замыканием, совпадающим с S^3 - это не важно. Тебе же нужно, чтобы осталась только одна свободная точка.
>> No.149240 Reply
>>149231
Тише,тише, значит ты просвещённый физик или матфизик :3
>> No.149241 Reply
>>149240
Тащемта биофизик почти гуманитарий
>> No.149244 Reply
>>149238
Ну вот, набор чисел, как и было сказано.
>> No.149253 Reply
что значит эта буква d? я не понимат
>> No.149254 Reply
>>149253
Зависит от того, где ты эту букву видишь.
>> No.149261 Reply
>>149237
>>149238
Нам на линейке тензора вводили как полилинейную функцию на векторах и ковекторах. Каковое определение инвариантно, элегантнее и вообще гораздо лучше, чем эти ваши "наборы чисел которые преобразуются так-то и так-то".
ещё один мимофизик
>> No.149263 Reply
>>149244
ГЕОМЕТРИЧЕСКИЙ ОБЪЕКТ же.
>> No.149265 Reply
>>149261
Так ведь это ваши наборы чисел, если я правильно понял. То есть, просто n-мерная таблица (пачка матриц) - набор чисел, как он есть.
не-физик
>> No.149266 Reply
>>149263
А что такое геометрический объект? Физики вообще любят выдумывать всякую ерунду, которая в математике не встречается. Например, "геометрическое место точек" или "изменяющееся число".
>> No.149267 Reply
>>149266
Физики норм. Просто они к математике имеют касательное отношение.
>> No.149268 Reply
>>149265
Ну вот вводить тензор как пачку матриц, это инженерский подход в худшем смысле этого слова. Наше теорфизическое образование от этого подхода весьма страдает.
>>149266
> А что такое геометрический объект?
Вероятно, математический объект, имеющий геометрическую природу. Что под этими словами подразумевается - загадка. Вероятно, что он (объект) нетривиально преобразуется при преобразованиях базиса.
>> No.149269 Reply
>>149268
Математический объект не может измениться при переходе к другому базису. Векторное пространство вообще не обязано априори обладать каким-то базисом, существование базиса - это теорема. Концепция "объект, меняющийся при замене базиса" - это концепция физиков.
>> No.149270 Reply
>>149269
Ну да, правильнее говорить "нетривиально преобразуется при вращении пространства". Потому что вращения, это уже группа каких-то преобразований векторного пространства. То есть - отображений. С преобразованиями самого пространства и преобразованиями координат (то бишь базиса) вообще в физике жуткий бардак. Их постоянно все путают.
>> No.149292 Reply
File: CLmNrfNUcAACgov.jpg
Jpg, 42.72 KB, 650×475 - Click the image to expand
edit Find source with google Find source with iqdb
CLmNrfNUcAACgov.jpg
Анон, а чем таким упороться для души, и чтоб не очень хардкорно? Вот, я когда-то давно был выперт с технаря и сейчас дизайнер, пенисы в иллюстраторе рисую, а вчера убирался, наткнулся на тетрать по матанализу, смотрю в нее и вообще ничего не понимаю, но думаю, классно все же, такие штуки понимал и решал, умный был у мамы.
>> No.149295 Reply
File: desicion.PNG
Png, 2.74 KB, 393×66 - Click the image to expand
edit Find source with google Find source with iqdb
desicion.PNG
Няши, можете объяснить баке в чём именно заключается метод наименьших квадратов на практике?

Не пойму, как решаются уравнения, которые они получили в конце метода парных сравнений. Гугл выдаёт не очень похожие примеры.

http://ecsocman.hse.ru/data/466/641/1219/ch6.pdf
>> No.149298 Reply
Убежище сосачерских математиков теперь будет здесь.
>> No.149299 Reply
Неприводимые представления над агл. замкнутым полем абелевой группы одномерны. Верно ли обратное?
>> No.149302 Reply
File: dMBJqRFc97s.jpg
Jpg, 57.13 KB, 775×583 - Click the image to expand
edit Find source with google Find source with iqdb
dMBJqRFc97s.jpg
>>149298
Сосач упал что ли?
>> No.149303 Reply
>>149299
Совершенно неверно. Самый простой пример: GL(2, F2) изоморфно группе перестановок S3: двумерное линейное пространство над F2 состоит из четырех точек, нулевая остается на месте при любом автоморфизме, а три остальные мы вольны переставлять как угодно. Если мы вложим Z/3Z в S3=GL(2, F2) как подгруппу, порожденную циклом длины три, то это и будет неприводимое неодномерное представление.
>> No.149313 Reply
>>149303
Не понял тебя. Ты опровергаешь не обратное к утверждению, а его само? В любом случае, F2 не алгебраически замкнутое поле.
>> No.149326 Reply
Поясните за неопределенный интеграл. А точнее, чем он так плох, что некоторые математики плюются и требуют его исключить из программы вузиков.
>> No.149327 Reply
>>149326
Просто далёк от практики - на реальных задачах первообразную можно найти чуть чаще, чем никогда. Только численные методы, только хардкор.
>> No.149329 Reply
>>149327
Я думаю, математики плюются еще потому, что по сути это и есть первообразная, которую непонятно зачем обозвали интегралом
>> No.149349 Reply
>>149326
> Я думаю, математики плюются еще потому, что по сути это и есть первообразная
Ну ещё и потому, что это устаревшее понятие, люди научились понимать это по-другому. Сейчас не за чем использовать лишний термин для формулы Ньютона-Лейбница (она же общая формула Стокса).
>> No.149548 Reply
чому фракталы считаются множествами дробно размерности?Чому у фракталов размерность определяется не как у других фигур?
>> No.149599 Reply
Здравствуй, доброанон. Посоветуй что-нибудь интересное по геометрии, топологии, вот этому всему.
>> No.149602 Reply
>>149599
А что тебе кажется интересным? Я вот от Куратовского-Мостовского офигенно прикалывался.
>> No.149606 Reply
>>149548
> у фракталов размерность определяется не как у других фигур
Так они принципиально отличаются от других фигур.
>> No.149652 Reply
Привет. Возможно я сейчас задам очень глупый вопрос, но всё же. Я тормоз, в этом году точно не пройду на всеросс, хоть и занимался математикой некоторое время (месяца 2), хотя к региональному этапу особо, честно говоря, не готовился. Значит ли это то что моё будущее как математика невозможно ? Математики - неолимпиадники - миф или реальность ?
P.s.Стоит упомянуть что я рассматриваю не при.мат, а научную деятельность
>> No.149656 Reply
>>149652
Олимпиадная математика к математике как науке не имеет вообще никакого отношения.
>> No.149658 Reply
>>149656
Однако я слышал, что в основном именно олимпиадники становятся выдающимися учеными. Реально ли стать учёным(если не выдающимся, то просто хорошим), если в школе с олимпиадами было туго ?
>> No.149659 Reply
>>149658
У меня нет статистики, собранной специально по этому вопросу. Среди цитируемых российских математиков олимпиадников я не обнаружил. http://dissertation-info.ru/index.php/-100-/179--100
>> No.149662 Reply
>>149658
Олимпиада - это решение более-менее стандартных задач за сжатые сроки. Ничего общего с наукой она не имеет.
>> No.149681 Reply
Ананасы, есть реквест, прошу помочь.
У меня есть знакомая, хочу сделать ей подарок, но на нём должно быть что-то, связанное с математикой. Сама она учится на математика в универе. Нужен какой-нибудь красивый график или вроде того. Ну не буду же рисовать синусы-косинусы, это уровень даунов. Лучше сразу несколько вариантов. Помогите, пожалуйста.
>> No.149687 Reply
>>149681
Какой еще график? Никому нахрен не нужен никакой график. Лучший подарок, который я видел, это годная книжка по математике (конкретно по тому разделу, который интересует жертву), на которой автор этой книжки написал пару предложений поздравлений и расписался. (Автора поймали после пары и полунасильно заставили это сделать).
>> No.149694 Reply
>>149687
Зачем дарить книжки, если они есть почти все бесплатные в интернете.
>> No.149695 Reply
>>144684
Что такое квазипозитивизм?
>> No.149696 Reply
>>149695
Спроси у тех, кто пользуется этим словом.
>> No.149701 Reply
>>149694
Книжки по техническим предметам намного проще читать так, чем с экрана.
>> No.149703 Reply
>>149687
Ну, типа декора на тему математики. По поводу книжек. Я понятия не имею, на какой там она ступени изучения. Предлагаешь наглым образом спросить, какие темы она уже прошла? Бред.
>> No.149706 Reply
>>149703
Нет смысла дарить книжки, в которых сам видишь фигу.
плакать опешила
>> No.149709 Reply
>>149706
ФОРМУЛЫ ПЛЕС
>> No.149711 Reply
>>149709
Неспособность обсудить содержание подаренной книжки девальвирует акт дарения в контексте неформальных межличностных взаимоотношений. Ты можешь не быть математиком сам и быть достойным в её глазах уважения в иных качествах, но не цепляясь за номинал профессии и генерируя поверхностные ужимки с покушением на прошаренность.
>> No.149782 Reply
File: question_000.PNG
Png, 138.27 KB, 1103×439
edit Find source with google Find source with iqdb
question_000.PNG
File: question_001.PNG
Png, 25.68 KB, 613×434
edit Find source with google Find source with iqdb
question_001.PNG

>>144684
>>144684
Здравствуй, кафедра!
Похоже, я тут всерьез и надолго. Готовлюсь к поступачу на лингвистику, дропнул техновуз. ВНЕЗАПНО понял, что без матстата и теорвера в лингвистике делать нечего (равно как и без дискретной математики и теории графов). Параллельно с подготовкой к ЕГЭ ботаю книжку Ширяева "Вероятность-1"

Сейчас разбираюсь с задачей подсчета сочетаний из М по n с повторениями - речь идет о эксперименте, когда из урны с M шаров достают шар n раз, причем возвращая вынутый шар перед изъятием следующего.

Я не совсем понимаю один момент, выделенный в тексте зеленым. Что значит a1<=a2<=...<=an? Ведь a1 может быть равно 4 шар, а an - 1-й шар. Или мы просто берем вариант (1,2,3,4) отбрасывая все эквивалентные ему потому что для удобства?
>> No.149784 Reply
File: question_003.PNG
Png, 3.97 KB, 543×138
edit Find source with google Find source with iqdb
question_003.PNG
File: question_002.PNG
Png, 38.47 KB, 1027×375
edit Find source with google Find source with iqdb
question_002.PNG

>>149782
Понял, как они развернули каждое слагаемое в формуле пикрилейтед (см. другой пикрилейтед). Непонятно только как они просуммировали все в количество сочетаний по n+1 из M+n. Тут какая-то формула? Если туплю, что смотреть из алгебры?
>> No.149787 Reply
>>149782
> Что значит a1<=a2<=...<=an? Ведь a1 может быть равно 4 шар, а an - 1-й шар.
Выборка неупорядоченная, тебе наплевать какой шар в ней первый, а какой 4й. Поэтому ты для удобства располагаешь их как хочешь.
>>149784
Развернули по треугольнику Паскаля C(n,k) = C(n-1, k) + C(n-1, k-1), соседние слагаемые с разными знаками схлопнулись.
>> No.149788 Reply
File: pilot_got_it.jpg
Jpg, 36.04 KB, 290×413 - Click the image to expand
edit Find source with google Find source with iqdb
pilot_got_it.jpg
>>149787
Спасибо большое, няша. Буду копать дальше же.
>> No.149843 Reply
Я вот заинтересовался такой темой. Код видео на Ютубе состоит из чисел, букв верхнего и нижнего регистра и дефисов. Используя приёмы из теории вероятности можно вычислить максимально возможное кол-во роликов на Ютубе. А что будет, когда все возможные комбинации символос исчерпаются? Ведь на место удалённых видео Ютуб новые не перегружает, те комбинации так и остаются неиспользуемыми. А люди всё равно ведь будут продолжать грузить на Ютуб всякий шлак. Так что же будет, когда всё возможные комбинации исчерпаются?
>> No.149844 Reply
>>149843
> Используя приёмы из теории вероятности
Ок, воспользуемся ими: 10 цифр, 26 заглавных букв и 26 строчных, итого 62. Ютуб сейчас 11 знаков юзает, что даёт 52 * 10 ^ 18 комбинаций. Это очень, очень, очень много, анон. Даже если заливать по миллиону роликов в секунду, уйдёт 165000 лет.
> что же будет, когда всё возможные комбинации исчерпаются
Люди вымрут и/или забьют на ютуб. А если не забьют - разрабы добавят ещё одну циферку, которой уж точно всем хватит (100млн лет по миллиону роликов в секунду).
>> No.149845 Reply
>>149844
P.S. Опечатался, 1.5млн лет, а не 165000.
>> No.149846 Reply
File: BHoSs_yYAOQ.jpg
Jpg, 110.44 KB, 717×960 - Click the image to expand
edit Find source with google Find source with iqdb
BHoSs_yYAOQ.jpg
Не знаю почему, но я в интернете практически не смог найти серьезных работ по изучению влияния математики на психику или работу мозга в целом.
Прозвучит банально, но у меня такой вопрос: Если тут те, которые были далеки от математики, а затем начали систематически заниматься ей? Мне интересно как она повлияла на ваше восприятие, принятие решений, общую собранность, etc. То есть, попробуйте проследить параллели между занятием этой наукой и вашей жизнью. Спасибо.
>> No.149894 Reply
Доброчан, тут такой вопрос.
Есть линейная рекуррентная последовательность, вроде фиббоначи. Я помню, можно было их как-то вычислять возводя матрицы. Подскажи пожалуйста.
Не подскажешь общую формулу для f(n) = k1f(n-1) + k2f(n-2) + k3*(n-3)...
Начальные значения тоже заданы, да.
>> No.149895 Reply
>>149894
Не уверен, что я тебя правильно понимаю, но для линейной последовательности, в которой каждый член зависти от n предыдущих, всегда легко составить матрицу вида
   (0 1 0 ... 0)
   (0 0 1 ... 0)
   (0 0 0 ... 1)
   (<строка коэффициентов>)
где верхние строки тупо двигают элементы столбца чисел из последовательности, а нижняя считает следующее число.
То есть для чисел фиббоначи это очевидно матрица вида
   (0 1)
   (1 1)
Каждым умножением на нее получается следующая пара чисел.
> Не подскажешь общую формулу для f(n) = k1f(n-1) + k2f(n-2) + k3*(n-3)...
Ты про производящие функции что-ли? Тогда что за общая формула? Вообще, про производящие функции неплохо изложено в "Лекциях по дискретной математике" Чашкина, или если по хардкору, то "Конкретная математика" Кнута.
>> No.149902 Reply
>>149895
Да! Спасибо, именно это и нужно было. Значит если мне нужно энное число, я беру (список нач. зачений)*(матрица коэффицентов)^n ?
>> No.149903 Reply
>>149902
Да, так. Только в >>149895-нотации это будет (матрица коэффицентов)^n * (столбец нач. зачений).
>> No.149910 Reply
>>149846
никак не повлияла
>> No.149985 Reply
Привет. И у меня есть ТУПОЙ ВОПРОС.

Есть универский туалет. Заходишь - справа зеркало, слева раковины, посередине кабинка, вход сбоку. За ней ещё две такие. У них один и тот же функционал. Вот только у дальней есть окно рядом, хотя изнутри кабинки окна не видно. Логичный вопрос: где чище?
Какую посетило меньшинство людей? В процентах — вообще идеально.

Я ЖЕ ВСЁ РАВНО ХОЧУ ЗНАТЬ ГДЕ ЧИЩЕ
КУДА БЛЯДСКОЕ СОЗНАНИЕ ПЕТУШКОВ ПОСПЕШИТ ЗАГНАТЬ СВОЙ ЗАД?

с:наркоман откройся
>> No.149986 Reply
>>149985
Ты ошибся кафедрой, анон.
>> No.149987 Reply
>>149985
Большинство будут идти в ближайший свободный, ибо хотят ссать. Особливо вонючая часть решит поближе к окну дабы вонь ушла быстрее. Туда же и курильщики.
В общем переменных не хватает! Но если "конь в вакууме" то у окна чище.
>> No.150028 Reply
Привет! У меня в шараге месяц как ввели новый предмет: "дифференциальные уравнения". Соль в том, что мне не повезло на нём быть ни разу и я не особо представляю о чём предмет. Посоветуйте, пожалуйста, годной литературы для самообучения. Желательно на русском или украинском языках. Спасибо!
>> No.150029 Reply
>>150028
Письменный, "Конспект лекций по высшей математике", часть II, глава X "Дифференциальные уравнения". Начни с этой главы.
>> No.150031 Reply
>>150029
Точнее, глава 1, верно? Ведь это %%http://nsportal.com.ua/book/3252%% та книга?
>> No.150032 Reply
>>150031
Сайт вирусный, предлагает мне скачать какой-то экзешник.
Вот нормальная ссылка: http://rghost.ru/6cqgd8mwG
>> No.150131 Reply
Кто-нибудь упарывает пенновский курс калькулюса на курсере? Давайте дружить.
>> No.150220 Reply
https://scrollback.io/coursera

Ну, если кто надумает - заходите, чятик без регистрации и смс.
>> No.150253 Reply
за эту просьбу не бейте, обоссыте лучше
Я сраный абитуриент, и мне, конечно же, надо осилить ЕГЭ, конкретно сейчас - задачи по геометрии. Из опыта знаю, что в задачах из учебника общего с текстом учебника в решении процентов двадцать, а у ЕГЭшных задач - и вовсе десять. Сейчас то, удаётся ли мне решить задачу или нет определяется чистым рандомом. Как вообще выучиться решать эти задачи? Какие есть пособия и методы?
>> No.150254 Reply
>>150131
И как тебе курс? Там же только лекции, да? Просто я плохо представляю себе, как можно учиться математике без семинаров, не решая разные задачки.
>> No.150271 Reply
>>150254
> И как тебе курс?
Охуенно. Лучший на курсере, как по мне. Лектор великолепен.
> Там же только лекции, да?
Нет. Там лекции; обязательный набор задач, решение которого влияет на итоговую оценку; challenge'вый набор задач; вики, служащая в качестве учебника\хэндаута лекций (тоже с задачками!). То есть в среднем ты 15 минут смотришь лекцию, потом джва часа решаешь задачки, переходишь к следующей.

Вместо семинаров там дискашн форумы, где можно спросить, если тебе что-то непонятно, и тебе даже ответят. Ну и вот я чатик запилил, но туда никто не приходит чего-то. Приходи ты :3
>> No.150272 Reply
>>150253
> Как вообще выучиться решать эти задачи?
Решать задачи.
> Какие есть пособия и методы?
Сканави.
>> No.150283 Reply
>>149299
Бамп бампыч
>> No.150309 Reply
>>144684
Аноняши, глупый вопрос - как улучшить навыки счета в уме?

Когда считаю на бумажке - все путем, как только считаю в уме что-то сложнее таблицы умножения - начинаю терять знаки, разряды, путать цифры и т.д.
>> No.150521 Reply
А вот пустое множество может являться элементом множества?
То есть, например, в таком
{1, 2, Ø}
множестве три элемента?
А в таком
{1, 2} - два?
>> No.150529 Reply
>>150309
Тебе оно надо? Нормальные ученые используют блокнот же.

>>150521
Может. Пустое множество - это полноценное множество, ничем не хуже прочих. Да, ты правильно посчитал элементы.
>> No.150714 Reply
File: jU6JCH0RDkY.jpg
Jpg, 95.91 KB, 1080×1350
edit Find source with google Find source with iqdb
jU6JCH0RDkY.jpg
File: KAXzjEozlP0.jpg
Jpg, 23.10 KB, 453×604
edit Find source with google Find source with iqdb
KAXzjEozlP0.jpg

>>150272
> Сканави.
Спасибо, держи тян, похожую на Цветаеву. и не тян ещё вот
>> No.150822 Reply
File: 71090d3f89a72c33bcc81ffbb7d87189.jpg
Jpg, 701.91 KB, 1287×1600 - Click the image to expand
edit Find source with google Find source with iqdb
71090d3f89a72c33bcc81ffbb7d87189.jpg
Няши, хочу угореть по математике, да вот незадача - в школе класса с 7 нехило так запустил это дело, хотя раньше учился на отлично, но кое-как вытянул на четвёрки. На уроках творился цирк, учителя не могли навести порядок, а дома я не занимался, ветер в голове и всё такое. Вот вопрос: стоит ли заново пробежать школьную программу с 7 класса чтобы восполнить недостатки, или можно сразу браться за литературу по интересам? Там скучно и многое мне в принципе понятно, только подводные камни не умею обходить. А может подскажете какой-нибудь годный вводный курс для таких как я великовозрастных балбесов? В этом году собираюсь поступать на математическую специальность по старым результатам егэ, знания за пару лет дуракаваляния как-то уже повыветривались.
>> No.150844 Reply
Меня давно мучает такой вопрос.
Есть формула разницы квадратов
a^2-b^2=(a-b)(a+b)
Можно ли её доказать используя только левую сторону тождества?
>> No.150847 Reply
>>150844
a^2 - b^2 = a^2 - b^2 + ab - ab = (a^2 - ab) + (ab - b^2) = a(a-b) + b(a-b) = (a-b)(a+b).
>> No.150865 Reply
>>144684
помогите пожалуйста, есть 3 частных решения ЛОДУ, y1=(x-1)^2, y2=x^2+1, y3=2x.Необходимо составить ЛОДУ(возможно более низкого порядка), имеющие такие частные решения. Помогите пожалуйста,объясните как его составить.
>> No.150876 Reply
>>150822
Ну, пока решай пробники. Просто готовься. За лето сможешь, я так думаю, освоить 7-11 классы с пониманием всех тем. А сейчас уже не успеешь. Просто задрачивайся. Заранее предупреждаю - будет больно.
>> No.151157 Reply
File: 12.png
Png, 5.94 KB, 602×331 - Click the image to expand
edit Find source with google Find source with iqdb
12.png
Привет, Анон.
Помоги советом.
Решил я вычислить некоторое число (отношение шанса встретить обычного покемона, к шансу встретить необычного) через множество проверок результата.
Но понял, что не знаю, как определить значение, которое можно считать результатом.
Хотелось бы узнать хотя бы первые две значащие цифры.
>> No.151245 Reply
File: 1nCwz-a6oag.jpg
Jpg, 65.92 KB, 604×604 - Click the image to expand
edit Find source with google Find source with iqdb
1nCwz-a6oag.jpg
Анон, такое дело: я тут писал рассказ. Просто начал писать ни о чём,захотелось по писать что-нибудь эдакое, поиграть словами, ничего серьёзного. Этакая немножко абсурдная история про молодого человека, который куда-то едет в автобусе.
Думаю, довольно-таки бессмысленно это пересказывать, а скинуть не могу, потому что ничего не готово и мне не ясно.
Но пишу я сюда вот почему: из-за автобуса, который едет куда-то откуда-то мне вспомнились школьные задачки. Потом главный герой начал думать о бесконечности в автобусном дискурсе. Я мне захотелось зашифровать в рассказ что-нибудь эдакое. То есть в названием сделать формулу/уравнение/any математическую операцию, а потом превратить рассказ в условие задачи. Проблема в том, что я НЕ ШАРЮ. И мне даже очень трудно объяснить, о чём я прошу.
В текст рассказа посреди путанных размышлений главного героя и психоделических описаний окружающего его мира зашифровано как бы условие задачки. Как те, по которым в школе нужно было составить, например уравнение и решить через это уравнение. И я был бесконечно признателен Анону, который бы предложил мне подходящую математическую операцию и разжевал её суть или хотя бы дал направление движения, чтобы я попробовал разобраться сам.
Кодовые слова: движение тела из точки А в точку Б, бесконечность, теория вероятности и всё, что может гармонично влиться в повествование которое я не скинул, лол.
>> No.151246 Reply
>>151245
Мне сразу в голову пришли ряды и сходимость по условию Коши. Хотя приложить это к рассказу непросто.
>> No.151247 Reply
>>151246
> критерию
Фикс
>> No.151267 Reply
Ищу учебник по математическому моделированию и оптимальному управлению. Темы - управляемость и наблюдаемость, автономный контроль, контроль линейных/нелинейных систем, оптимальное управление. Можно на инглише. Большой плюс если будет расжевываться решительно все, потому что с математикой у меня не ахти.
>> No.151269 Reply
File: 14503441152500.gif
Gif, 5704.69 KB, 255×255 - Click the image to expand
edit Find source with google Find source with iqdb
14503441152500.gif
>>144684
Суп, матемач!
Вот есть ведь R^3-редакторы типа "3ds max". А есть R^4 редакторы?
Ну, пикрелейтед в чем-то же нарисовали! Да и, помню, видел когда-то в ТыТрубе одно видео с многомерным редактором, но названия не помню.
>> No.151286 Reply
>>151269
Твой гиф - рендер 3д обьекта, который скриптом генерируется из 4-х мерной формулы. Такие математические штуковины руками не рисуют - задают формулы, которые все делают автомагически.
> одно видео с многомерным редактором
А вот на это я бы посмотрел.
>> No.151353 Reply
File: Снимок-экрана-(269).png
Png, 76.72 KB, 599×588 - Click the image to expand
edit Find source with google Find source with iqdb
Снимок-экрана-(269).png
Читаю "Что такое математика". Застрял на первых же упражнениях - прогрессии, доказательства равенств. Как вообще это решать? И как понять что решил правильно?
>> No.151354 Reply
>>151353
Прогоняй брутфорсом и поглядывай на численную сходимость графика двух половинок равенства.

прогулял математику
>> No.151355 Reply
>>151354
Вряд ли автор подразумевал чтобы я делал так.
>> No.151356 Reply
>>151353
Упражнение 1.
Пункт 1. Для n=1 формула верна, ибо 1/2=1/2.
Пусть формула верна для n. Тогда, так как n/(n+1) + 1/(n+1)∙(n+2) = (nn + 2n + 1)/(n+1)∙(n+2) = (n+1)/(n+2), формула верна для n+1. По индукции, формула верна для всех n.
>> No.151364 Reply
>>144684
Братки, у меня очень необычный вопрос. Я пытаюсь написать небольшое сочинение по матану: вводный учебник, который знакомит адептов с основами учения и подает основные идеи.
И первым делом я хочу подобрать какие-то нибудь бытовые примеры интегрирования и дифференциирования. Показать идею операции. Но так, чтобы легко было видеть, что операции обратны друг к другу.
Ситуация, кроме того, осложняется ещё тем, что понимать их можно двояко: скажем, интегрированием называют и нахождение одного числа (определённое) и функции (неопределённое). И надо подобрать пример достаточно хороший, чтобы обозначить простоту операций и не запутать читателя
>> No.151365 Reply
>>151356
А как так же девятое например решить? Сколько сижу уже и решительно не понимаю, как привести всё к нужному виду, делаю кучу преобразований, но смысла в итоге от них совсем нет, только путаницы больше.
>> No.151366 Reply
>>144684
Анон, вопрос от незнайки про четырёхмерное пространство.

Часто в качестве демонстрации показывают проекцию тессеракта, который ещё и выворачивается. А почему я не видел аналогичную проекцию куба на плоскость? Это бы многое прояснило для интуитивного понимания.
>> No.151372 Reply
>>151366
Просто включи любое видео с вращающимся "трёхмерным" кубом.
>> No.151377 Reply
File: cube-project.png
Png, 267.32 KB, 540×540 - Click the image to expand
edit Find source with google Find source with iqdb
cube-project.png
>>151366
Получите ориджинал контента.
>> No.151407 Reply
>>151366
> А почему я не видел аналогичную проекцию куба на плоскость?
А что же ты видел на плоском экране монитора?
>> No.151414 Reply
Позволю себе поправить >>151407-куна: проекция не несет в себе информации о светотени и глубине пространства. Проекция - это несколько простых математических линий. А на дисплее он видит именно закос под реальное 3-мерное пространство, с освещением и гранями.
>> No.151422 Reply
>>151365
Бамп вопросу. У меня чувство что я скоро с этими доказательствами свихнусь совсем.
>> No.151423 Reply
>>151422
Читай то, что тебе интересно. В книге, которую ты читаешь, есть строчки поинтереснее этого упражнения номер девять.
>> No.151432 Reply
File: randbitmap-rdo.png
Png, 67.32 KB, 512×512 - Click the image to expand
edit Find source with google Find source with iqdb
randbitmap-rdo.png
Давайте обсудим кое-что.
Тема поднималась в треде в /b в связи с теоремой Гёделя, я описал свои представления, но оказывается, есть в чём ещё разобраться.
>>b/4198689
У нас есть построения. Есть свойства таких построений, и вот это уже туманное понятие.
Есть построения, которые выражают свойства тех построений. Наверное, не обязательно, чтобы те и эти построения были одними и теми же построениями?
Надо бы прояснить понятие свойства построений и найти широкую формулировку исследования и заключения.
>> No.151433 Reply
>>151432
Что именно тебя интересует и в каком объёме ты владеешь логикой? Ты читал книжку "Введение в метаматематику" Клини?
>> No.151436 Reply
>>151433
Выражался в терминах, которые использованы в txt.
> Ты читал книжку "Введение в метаматематику" Клини?
Полностью не читал. Да и то, что касается вопроса, там наверняка изложено туманнее, чем это сделал я в txt. Единственное излишество, которое я вижу сейчас, это обращение к натуральным числам, но оно необходимо было чтобы можно было рассчитывать на понимание написанного.
> Что именно тебя интересует
> > Надо бы прояснить понятие свойства построений и найти широкую формулировку исследования и заключения.
>> No.151454 Reply
>>151432
Пока мне кажется, что не стоит делить построения на такие-то и такие-то.
Если для частного понятия построения присутствуют изначальные построения и некие действия, то существует взаимооднозначное соответствие между любыми такими построениями и любыми такими построениями, в частном понятии которых также присутствуют изначальные построения и некие действия.
>> No.151576 Reply
>>151353
Смотрю я на такие "вводные курсы" и блевать хочется. Проклинаю всех авторов учебников по математике, самые убогие учебники эвар.
>> No.151602 Reply
>>151576
Кажется, я тебя понимаю.
Ну и вообще, учебники — шлак.
>> No.151631 Reply
File: 14606444825320.jpg
Jpg, 1098.40 KB, 3840×2160 - Click the image to expand
edit Find source with google Find source with iqdb
14606444825320.jpg
Самостоятельно прохожу школьный курс математики (19 лвл), и уже дошел до логарифмов, но на прошлых темах было много заданий, которые я не смог решить, хотя большинство всё же было решено, но вот на пике выписана часть из них, по разным темам. Может анон сможет найти решение хотя бы первых двух, ибо эти задания были в начале книжке, и я до сих пор не могу их решить.
   Ну к примеру, 29, допустим понятно, что корень из 60, это 2 корня из 15, привели к общему. Но что делать с первым и последним, ведь их невозможно привести. И в других примерах, которые вроде бы не сложные, но "не получается". Учебник Киселева, написанный в конце 19-го века. Спасибо.
>> No.151649 Reply
File: paper-day1-bg-Rus...
Pdf, 0.29 KB, 595×842
paper-day1-bg-Russian.pdf
File: paper-day2-bg-Rus...
Pdf, 0.28 KB, 595×842
paper-day2-bg-Russian.pdf

Посоны, я вам задачи с олимпиады для девочек принес. По-моему, годнота.
>> No.151650 Reply
>>151631
Не понял, в чем сложность? 5/3=25/15; 3/5=9/15, итого
(10+3)/[15]+3[15] = 13/[15]+3*15/[15]=58/[15], если я в арифметике не накосячил.
Со следующим аналогично.
Дальше перенести корень в правую часть, х - в левую, возвести все в квадрат, не забыть отсеять корни, при которых под корнем отрицательно число. Последнее аналогично.
Судя по тому, что ты и системы не можешь решить, у тебя какие-то проблемы с домножением левой и правой частей уравнений (а также числителя и знаменателя дробей) на нужное значение. Не бойся его. Бери и умножай.
>> No.151653 Reply
Можно ли перенести неравенство Бернулли на вещественные числа, и если можно, то как ?
Спасибо
>> No.151656 Reply
Я глянул недавно впервые программу Вербита. Почему там нет дискретки (если я чего - то не понимаю, не ругайтесь, я школьник и во всех этих умных словах понял только пункты про матшкольника)? В наше время, с компьютерными технологиями это же невъебенно важный раздел
>> No.151663 Reply
>>151656
Потому что никакой дискретки не существует, это фикция. Неучи взяли три статьи из википедии, обозвали "дискретная математика", прочитали их - называют себя йоба-учоными теперь.
>> No.151665 Reply
>>151663
А как же всякие теории алгоритмов, всякие графы и это всё ? Оно присутствует в программе Вербитского ?
Алсо в универах же есть такой предмет. И там преподают, как по мне, совсем недурные вещи(например на ФКН ВШЭ в сильной группе там нехилые задачи).
>> No.151666 Reply
File: classifications2010.pdf
Pdf, 1.34 KB, 877×1076 - Click the image to get file
classifications2010.pdf
>>151665
> Алсо в универах же есть такой предмет.
Прикреплейтед - настоящая классификация разделов математики, MSC 2010. MSC существует с 1940 года, её составляют самые уважаемые математические организации, так что она заслуживает доверия.
> Оно присутствует
Нет.
>> No.151671 Reply
>>151666
> Recreational mathematics
ОК.
Но вообще, Theoty of Computing у нас обычно и называется дискретной математикой: теория сложности, автоматы, формальные языки. А прямо за ней, сюрприз, Discrete mathematics in relation to computer science.
>> No.151674 Reply
>>151666
Тут есть теория графов, комбинаторика, теория чисел. Это всё не части дискретной математики?
>> No.151675 Reply
>>151674
Да части, конечно. enwiki://Discrete_mathematics
Твой собеседник вербитофан, ему важно в слова играть, а не заниматься математикой.
>> No.151676 Reply
>>151675
Ясненько.
А почему всё-таки в той программе нет дискретной математики?
>> No.151677 Reply
>>151676
Она написана для того, чтобы как можно быстрее сделать человека компетентным математиком-исследователем - например, чтобы человек как можно быстрее понял доказательство теоремы Атьи-Зингера об индексе. То, что ты называешь "дискретной математикой", не нужно на этом пути.

>>151675
> в слова играть, а не заниматься математикой
Занимательное противопоставление.
>> No.151678 Reply
>>151677
А для построения SMT-решателя нужно. Дальше что?
>> No.151679 Reply
>>151678
Кто-то занимается математикой, кто-то строит SMT-решатели. Та программа - для математиков.
>> No.151681 Reply
>>151679
> не нужно на этом пути
Теория алгебр Ли не математика?
>> No.151682 Reply
>>151681
Первый курс, последний пункт.
>> No.151683 Reply
>>151678

О чём вообще спор, посоны?

Ой, оказывается, есть люди, которым интересен computer science, есть люди, которые хотят в современную топологию, есть люди, которым математика нужна, чтобы числодробилки писать, а есть люди, которые занимаются всякими оценками всяких чисел Рамсея.

Разные люди занимаются разными вещами. Ох, нихуясебе, новость! Не может такого быть! Кто-то, определённо, неправ!
>> No.151684 Reply
>>151679
Задача выполнимости формул в заданной системе предикатов - не математика? Ну охуеть теперь.
>> No.151687 Reply
>>151684
Нет, это что-то между логикой и информатикой.
>> No.151689 Reply
>>151682
Ясно
>> No.151690 Reply
>>151683
Однако некоторые базовые вещи все математики должны знать. А та программа составлена из расчёта, что все математики должны как можно быстрее понять доказательство теоремы Атьи-Зингера об индексе и на ней свет клином сошёлся. Так что там даже школьники не проходят дискретной математики.
>> No.151692 Reply
>>151690
Что значит "проходят дискретную математику"? Какой именно перечень фактов подразумевается под "дискретной математикой"?
>> No.151695 Reply
>>151690
> Однако некоторые базовые вещи все математики должны знать.
Это какие?
И кому они должны?

А не должны ли они. случайно, ещё уметь собирать кубик рубика на спор, играть в Го, программировать на Си и прочитать и осознать Ландавшифца? Ну, так, потому что это базовые вещи, которые каждый должен знать.
>> No.151696 Reply
>>151695
Если человек является математиком, то он понимает теорему Атьи-Зингера об индексе.
Следовательно, если человек хочет считаться математиком, он должен по меньшей мере знать всё то, что требуется для доказательства этой теоремы.
>> No.151697 Reply
>>151695
> кому они должны?
> должен понимать теорему Атьи-Зингера
>> No.151698 Reply
>>151697
Математик, который не знает теорему Атьи-Зингера, - это как программист на Си, который не знает libc. Смешное явление.
>> No.151699 Reply
Полистал первую книжку хваленых Бурбак. Надеюсь после этого мои учебники покажутся легкой утреннеё прогулкой по парку.
>> No.151716 Reply
>> No.152195 Reply
File: 17955537_1374691480086.jpg
Jpg, 1975.13 KB, 4000×4000 - Click the image to expand
edit Find source with google Find source with iqdb
17955537_1374691480086.jpg
>>144684
Перельманы, а почему нет адвайс-листа с книгами по математике как у кодеров? Может запилим? или я его прослоупочил


Password:

[ /tv/ /rf/ /vg/ /a/ /b/ /u/ /bo/ /fur/ /to/ /dt/ /cp/ /oe/ /bg/ /ve/ /r/ /mad/ /d/ /mu/ /cr/ /di/ /sw/ /hr/ /wh/ /lor/ /s/ /hau/ /slow/ /gf/ /vn/ /w/ /ma/ /azu/ /wn/ ] [ Main | Settings | Bookmarks | Music Player ]